Cuestiones
ayuda
option
Mi Daypo

TEST BORRADO, QUIZÁS LE INTERESEEXAMEN COMPLEXIVO ONLINE 2021

COMENTARIOS ESTADÍSTICAS RÉCORDS
REALIZAR TEST
Título del test:
EXAMEN COMPLEXIVO ONLINE 2021

Descripción:
EXAMEN COMPLEXIVO ONLINE UNIFICADO

Autor:
AVATAR
Jonathan Guamán
(Otros tests del mismo autor)


Fecha de Creación:
07/01/2021

Categoría:
Idiomas

Número preguntas: 410
Comparte el test:
Facebook
Twitter
Whatsapp
Comparte el test:
Facebook
Twitter
Whatsapp
Últimos Comentarios
No hay ningún comentario sobre este test.
Temario:
FIRST TERM: 1. En fonología, _____ se refiere a los cambios de sonido mediante los cuales una consonante cambia su tipo de voz de voz a voz, o viceversa, debido a la influencia de su entorno fonológico. a. Voicing b. Aspiración c. Modo de articulación.
2. Si estaba leyendo un texto desconocido, ¿Qué claves textuales lo ayudarían a determinar el período de tiempo aproximado en el que se estableció la historia? a. Qué edad tiene el libro, los nombres de los personajes y el tipo de comida que se come b. Realmente no hay forma de determinar el período de tiempo. Solo tienes que esperar a que el autor lo revele c. Ropa, patrones de habla, vocabulario y comportamiento.
3.The notion of this discipline is necessary for areas of cross-cultural communication, for correct use of language in general, because it is considered the goal of language education, and it is necessary element in efficient classroom practice. It refers to: a. Communicative Competence b. Linguistic competence c. Universal Grammar theory.
4.Choose the option that matches the information below. This term refers to the language used by people who work in a particular area or who have a common interest. It can be used to express ideas that are frequently discussed between members of a group, though it can also be developed deliberately using chosen terms. a. Slang b. Jargon c. Lexical competence.
5.Choose the letter of the answer that best completes the following sentence. They _____________ make sure patients’ records are accurate. This is one of their responsibilities. a. have got b. have got to c. must to.
6. Choose the correct answer. Syntactically, ____ is a large construction expressing full meaning and forming a grammatical unit. It has two main constituents: an NP that occupies the subject position, and a VP that functions as the predicate: a. a clause b. an statement c. a sentence.
7. Read the editorial. Then use the information to complete the activities that follow. Creating a Smartphone Monster 1 I remember it as if it were yesterday—walking through the front door of my mother’s house and shouting, “It’s here!” Mom poked her head out of the kitchen, rolled her eyes when she saw the smartphone packaging, and then disappeared again. Not the best start, I’ll admit. Still, I had done my research, and all the latest studies showed that it was very helpful for seniors to learn how to use new technology devices. So, I decided I would be the one to make it happen! 2 Hours later, I finally ensured that her email account and contacts were set up. I did not relent until we reviewed a few apps and figured out the all-important ringtone. It was a long night, but even as I left, I knew that my mother had only retained half of what I had told her (if I was lucky). It’s not that there’s anything wrong with her. In fact, she’s one of the smartest people I know. It’s just that she came from a generation that didn’t have mobile phones at all, let alone smartphones. This influx of information was all very new and confusing. 3 After a few days of phone calls, her anxiety slowly diminished until my mother finally stopped asking how to do every little thing on her phone. It wasn’t until I had the urge to go shopping with her that I became conscious of the monster I had created. You see, Mom was now a smartphone addict. She used her phone for everything. Should I wonder aloud about anything at all, she would twitch excitedly and say, “They have an app for that.” She was also suddenly a compulsive email checker—the same woman who used to go days at a time between email responses. 4 But her biggest dependency was her texting. My jaw dropped when I came out of a store and found her fingers moving at the speed of light. “Mom!” I called to her, but she was “in the zone” and the temptation to continue texting was apparently too great. (I learned later that she was discussing a recipe with Aunt Lucy.) Trying to get her attention, I reached for her phone, triggering what could only be described as the mother-daughter cat fight of the century—right in the middle of a shopping mall. All the while, she clutched her smartphone as if her life depended on it, screaming “Help!” at the top of her lungs. 5 Later, that evening, I told my husband about her smartphone dysfunction and how we would have to slowly wean her away from using it. Hopefully she wouldn’t have any withdrawal symptoms. I watched him literally cry from laughing so hard. “Don’t you see the humor in all of it?” he finally asked when he saw I wasn’t smiling. I think my silence answered his question. 1 app: application, or program for a mobile device According to the information, which of the following statements is false? a. The author believes her mother should not use her smartphone as much as she does. b. The author’s mother never really understands how her smart phone works. c. The author’s mother does not want the smartphone when the author gives it to her.
8. Voicing refers to: a. activity of the vocal cords b. movement of tongue inside the vocal tract c. audible escape of air following some consonant sounds.
9. Read and choose the correct definition of Instrumental Motivation. a. It is based on the desire to know more about the culture and community of the target language group and even a desire to be more like members of that group. b. It is essentially practical, such as need to learn the language in order to get a better job. c. It consists of a set of predetermined categories used to record and describe teaching and learning behaviours.
10. Choose the correct option for each sentence. a. Janice must not understand me when I said I wouldn't be home for dinner. b. Janice must not have understood me when I said I wouldn't be home for dinner. c. Janice mustn't have understood me when I said I wouldn't be home for dinner.
11. ____ Leslie had many birth defects. He had to have his eyes removed and did not stand until he was 12 years old. He had difficulty doing the same things that other people do not think about, such as swallowing food. Yet, Leslie is able to play a piece of music after hearing it only once. He is a true savant. a. Many people think Leslie Lemke is a musical genius. b. Leslie Lemke is a musical genius who has many disabilities. c. Leslie Lemke was adopted when he was six months old.
12. Read the editorial. Then use the information to complete the activities that follow. Creating a Smartphone Monster 1 I remember it as if it were yesterday—walking through the front door of my mother’s house and shouting, “It’s here!” Mom poked her head out of the kitchen, rolled her eyes when she saw the smartphone packaging, and then disappeared again. Not the best start, I’ll admit. Still, I had done my research, and all the latest studies showed that it was very helpful for seniors to learn how to use new technology devices. So, I decided I would be the one to make it happen! 2 Hours later, I finally ensured that her email account and contacts were set up. I did not relent until we reviewed a few apps1 and figured out the all-important ringtone. It was a long night, but even as I left, I knew that my mother had only retained half of what I had told her (if I was lucky). It’s not that there’s anything wrong with her. In fact, she’s one of the smartest people I know. It’s just that she came from a generation that didn’t have mobile phones at all, let alone smartphones. This influx of information was all very new and confusing. 3 After a few days of phone calls, her anxiety slowly diminished until my mother finally stopped asking how to do every little thing on her phone. It wasn’t until I had the urge to go shopping with her that I became conscious of the monster I had created. You see, Mom was now a smartphone addict. She used her phone for everything. Should I wonder aloud about anything at all, she would twitch excitedly and say, “They have an app for that.” She was also suddenly a compulsive email checker—the same woman who used to go days at a time between email responses. 4 But her biggest dependency was her texting. My jaw dropped when I came out of a store and found her fingers moving at the speed of light. “Mom!” I called to her, but she was “in the zone” and the temptation to continue texting was apparently too great. (I learned later that she was discussing a recipe with Aunt Lucy.) Trying to get her attention, I reached for her phone, triggering what could only be described as the mother-daughter cat fight of the century—right in the middle of a shopping mall. All the while, she clutched her smartphone as if her life depended on it, screaming “Help!” at the top of her lungs. 5 Later, that evening, I told my husband about her smartphone dysfunction and how we would have to slowly wean her away from using it. Hopefully she wouldn’t have any withdrawal symptoms. I watched him literally cry from laughing so hard. “Don’t you see the humor in all of it?” he finally asked when he saw I wasn’t smiling. I think my silence answered his question. 1 app: application, or program for a mobile device The editorial was written to explain how smartphones are ____. a. easy to get addicted it b. difficult to understand c. necessary for everyone .
13. When students deal with the so called Notice-Pattern-Contrast Feeling model answering to key questions: What, Who, When, Where, How, Why, and What if? in a poem, they are actually dealing with a model in which they discover: a. How to write a text b. How to interpret a text c. How to approach a text.
14. The Affective Filter hypothesis embodies Krashen's view that a number of 'affective variables' play a facilitative, but non-causal, role in second language acquisition. Which are these variables? a. Personality, learning strategies and motivation b. Motivation, self-confidence and anxiety. c. Self-confidence, age and anxiety.
15. ____ is a kind of secret language, for example, words are created in a kind of code language by “shifting” vowel and consonant sounds. a. Jargon b. Slang c. Pig latin.
16. Elige la opción correcta para cada oración a. Será mejor que nos pongamos en marcha pronto, o no llegaremos al juego a tiempo. b. Mejor nos ponemos en marcha pronto, o no llegaremos al juego a tiempo. c. Mejor nos ponemos en marcha pronto, o no llegaremos al juego a tiempo.
17. The following sentence contains a mistake. Choose the option that has the same sentence written correctly. Remember to check punctuation. If I do not like something I bought online, then I returned it. a. If I do not like something I bought online then I returned it. b. If I do not like something I bought online, then I return it c. If I do not like something I bought online then I return it.
18. Read the following statement and choose the correct alternative to complete it. A ____________ usually presents one main idea. The sentence that states the main idea is called a ____________. The following sentences support the topic sentence with ____________ like reasons or examples. a. topic sentence / supporting detail / questions b. paragraph / topic sentence / details c. sentence / draft / conclusion.
19. Which of the following is not an original contributor to the English language? a. French b. Russian c. Latin.
20. Read this introduction and choose the thesis statement. In our local area, there are many community service projects that young people can take part in. Of the opportunities which exist, some of the most important projects are those focusing on environmental protection. This is because raising awareness of environmental issues is one of the most important issues of our times. Beach cleanups are one way that young people can help the environment, but there are many other opportunities, too. a. Of the opportunities which exist, some of the most important projects are those focusing on environmental protection. b. Beach cleanups are one way that young people can help the Beach cleanups are one way that young people can help the environment, but there are many other opportunities, too. c. In our local area, there are many community service projects that young people can take part in.
21. Choose the correct option to complete the paragraph. When the substitute teacher entered the class yesterday, she didn't introduce herself so Maria said, "Who are you?" It was strange that Maria had to ask ________. a. who the substitute is b. who the substitute was c. who was the substitute.
22. It is a retelling activity which displays something (a story) that happened. Remember that it is not conceived as the story itself but rather the telling of the story which could be written or spoken. a. Interpretation b. Speech c. Narrative.
23. Which of the following is the term used to describe when some features of learner language stop changing over time? a. Linguistic freezing b. Fossilization c. Grammar Translation.
24. Please choose the correct answer from the choices presented. In literature, three main genres are distinguished: poetry, novel and drama; each of these main genres can be sub-divided into smaller genres like lyrics, sonnets and ballads for poetry. With these ideas in mind, choose the best description of the work titled “Robinson Crusoe” written by Defoe in the 18th century. a. Epic poem b. Narrative novel c. Comedy .
25. It studies the nature and limits of human ability to produce speech sounds and describes the way these sounds are delivered. Besides, it studies the production of speech sounds. a. Acoustic phonetics b. Articulatory phonetics c. Mixed-ability phonetics.
26. This prosodic element refers to the use of melodic pattern of a language and includes rise and fall of the voice when speaking. It is true about: a. Intonation b. Pitch c. Accent.
27. ____ is the alternate use of two or more variables in the same conversation or sentence. It is traditionally considered a spoken, informal mode of speech but can also be found in writing. a. Accent b. Dialect c. Code-switching.
28. Identify the words that are pronounced with aspirated initial phonemes: a. vowel, train, plan, clay b. steak, blow, crab, frog c. take, cat, keep, pale.
29. Apply syntactic analysis on the sentence: Meg told Jane that her father was sick. a. The complementizer THAT can easily be omitted without changing the meaning of the sentence. b. The complementizer THAT introduces additional information to the sentence. c. The complementizer THAT cannot be delated because the meaning of the sentence could change. .
30. Lea la analogía y responda la siguiente pregunta. "Bloguear es otro ejemplo que se compara mejor con escribir una entrada de diario personal y publicarla para que el mundo la lea". ¿Qué punto está tratando de hacer el autor? a. una entrada de diario personal y el mundo b. La información personal se revela a través de los blogs. c. El mundo es mucho más grande de lo que crees.
31. If you were preparing to read a Historical piece of literature, which of the following would be the best question to ask before starting to read? a. Where and when does the story take place? b. How much wealth and social status does the main character have? c. What is the main characters relationship to the culture?.
32. Which of these are not considered to be a part of language? a. Self-talk and deep thoughts b. Body language and facial expressions c. Using Snap Chat on your phone.
33. Read the outline of a cause-and-effect essay. Then answer the questions. 1. Introductory Paragraph Thesis statement: “Starting a blog about a topic of interest can have many benefits, including organizing personal ideas and information, making friends whom share the same interest, and learning and sharing new information with others over time.” 2. Body paragraph 1: organizing personal ideas and information example: gardening 3. Body paragraph 2: making friends who share the same interest meeting people through comments 4. Body paragraph 3: learning and sharing new information with others over time solving problems through blog discussions 5. Concluding Paragraph What is the essay about? a. the benefits of gardening b. the benefits of searching the Internet c. the benefits of having a blog.
34. A standard for teaching pronunciation to non-native speakers of English is: a. Standard British b. Received, or BBC c. Standard American .
35. Cuando un hablante adulto simplifica, modifica y adapta deliberadamente su discurso de acuerdo con la edad, el estado, la educación o los antecedentes culturales de un compañero de habla, o mezcla dos o más idiomas en el mismo discurso, este fenómeno se llama: a. comportamiento del lenguaje b. non- standard language c. code switching.
36. Choose the option that contains the correct indirect form of the sentence below. Mary said, “I do not have any money now.” a. Mary said that she has not had any money that day. b. Mary said that she did not have any money now. c. Mary said that she did not have any money then. .
37. Please choose the correct answer from the choices presented. It implies the possible application of interpreting by means of performance as well as analysis. The resulting outlook may be an analysis of a text, a creative writing, or a critical reading. a. Speech b. Interpretation c. Narrative.
38. Choose the option that is not correct. a. Phonology studies human speech sounds, how these sounds are produced and methods for their classification. b. Phonology describes the way sounds function within a given language or across languages to encode meaning. c. Phonology is both physical and meaningful. .
39. Read this introduction and choose the thesis statement. Many people disagree with the idea of requiring students to participate in mandatory community service projects. They argue that such projects are ineffective because they are often short term. Moreover, they say that these projects may even discourage young people from volunteering of their own accord in the future. In this essay, I will explain why these objections are incorrect, and will provide a number of arguments in favor of mandatory community service projects. a. Many people disagree with the idea of requiring students to participate in mandatory community service projects. b. In this essay, I will explain why these objections are incorrect, and will provide a number of arguments in favor of mandatory community service projects. c. They argue that such projects are ineffective because they are often short term.
40. Read the paragraph. The underlined supporting detail is weak. For this underlined sentence, choose the best replacement that provides facts, examples, or explanations. Paragraph: Mount Everest has become an overcrowded, polluted tourist location. Too many people visit it every day. Many of these tourists are inexperienced climbers and do not belong there. The pollution at the site is getting worse all the time. Tourists leave behind items from their journey. The problem is that the tourists do not respect their environment. The government of Nepal plans to solve this problem. a. Tourists leave behind things they brought with them on their climb. b. Tourists leave behind items and other things from their trip. c. Tourists leave behind oxygen tanks, tents, and even food wrappers. .
41. Modifiers can be obligatory or optional. It depends on the role each constituent has within a sentence to determine the types of modifiers it includes. For instance, in the sentence: “Michael and Ana are considering to visit New York next month” New York works as a part of the… a. adjunct of the verb visit b. subordinator to link the verb and the object. c. complement of the verb visit.
42. What is the difference between Canon and Classic? a. Canon is a work by a famous author, Classic is on old book b. Canon is a great work that is worthy to be read by everyone, Classic is popular across cultural groups. c. Canon is an old written story, Classic is written in old English.
43. Elija las dos palabras que representan pares mínimos: a. nudo de madera - no enviado - olor a lago - rezar b. neblina - hileras de llamas - rosa oye - cabello rubio c. el ruido de la nariz podría, buena voluntad, buena cabra.
44. Some theories, models, and perspectives have different points of view of second language acquisition. Which of the following options refers to the statement that “Knowledge is constructed through interaction and collaboration with others”? a. The sociocultural perspective b. The behaviorist perspective c. The cognitive perspective.
45. Syntax establishes the use of three different types of sentences: simple, complex and compound. In regards to compound sentences, they are formed by two clauses linked by a connector. Hence, their structure would be… a. One dependent clause, a subordinator, and one independent clause b. One independent clause, a coordinating conjunction, and one dependent clause c. One independent clause, a coordinating conjunction, and one independent clause.
46. Read the paragraph. Then answer the questions that follow. _______. The place is never crowded, but the service is still terrible. It seems like the staff has better things to do than wait on customers. Once I waited ten minutes just to buy a slice of pizza, and I was the only person in the whole store! I recommend staying away from Billy Bob’s Pizzeria. Which is the topic sentence of this paragraph? a. Billy Bob’s Pizzeria has the worst pizza. b. Billy Bob’s Pizzeria is an unpopular place to eat. c. Billy Bob’s Pizzeria has terrible customer service.
47. Read the information below and complete the sentence. Clarification requests, Elicitation and Repetition are some techniques that teachers use as… a. Instructional approach. b. Observation schemes. c. Corrective feedback.
48. It is concerned with the physical properties of speech sounds such as: their physiological production, acoustic properties, auditory perception, and neurophysiological status. a. Voicing b. Phonology c. Phonetics.
49. Please choose the correct answer from the choices presented. It implies the possible application of interpreting by means of performance as well as analysis. The resulting outlook may be an analysis of a text, a creative writing, or a critical reading a. Narrative b. Interpretation c. Speech.
50. Choose the best topic sentence to complete the following paragraph. Generally speaking, zoos these days are losing popularity. Many people feel that it’s unfair to lock up wild animals to entertain humans. ________________________________ . Some zoos go to great lengths to build a good environment for its animals. The good zoos treat their animals well and provide opportunities for the public to learn about the natural world. The best zoos protect endangered species and raise awareness about animals and their habitats. a. Zoos are horrible places that should be shut down because it’s not humane to lock up wild animals. b. Zoos perform a valuable service to society and should be part of every person’s childhood memory. c. Zoos that do their best to make a pleasant habitat for the animals can be a pleasant place to visit. .
51. Read and choose the correct option. Chomsky´s concluded that children´s minds are not blank slates to be filled by imitating languages they hear in the environment. Which perspective claims the previous information? a. The Behaviorist perspective b. The Innatist perspective c. The developmental perspective.
52. It is essentially practical, such as need to learn the language in order to get a better job. Scheme. a. It consists of a set of predetermined categories used to record and describe teaching and learning behaviors b. It is based on the desire to know more about the culture and community of the target language group and even a desire to be more like members of that group. c. It is essentially practical; such as need to learn the language in order to get a better job.
53. Choose the correct option to complete the following paragraph. Nowadays, one of the most______ items found in the home is the sticking plaster. Protecting a cut by covering the affected area with a piece of material that sticks to the skin may seem a rather_____idea, so it is perhaps surprising to learn that the plaster was not _____until about ninety years ago. a. general, clear, invented b. common, obvious, invented c. frequent, evident, realized.
54. The following sentence contains a mistake. Choose the option that has the same sentence written correctly. Remember to check punctuation. a. If I do not like something I bought online then I returned it. b. If I do not like something I bought online then I return it c. If I do not like something I bought online, then I return it.
55. Lea la analogía y responda la siguiente pregunta. "Bloguear es otro ejemplo que se compara mejor con escribir una entrada de diario personal y publicarla para que el mundo la lea". ¿Qué punto está tratando de hacer el autor? a. El mundo es mucho más grande de lo que crees. b. una entrada de diario personal y el mundo c. La información personal se revela a través de los blogs.
56. Choose the correct option to complete the paragraph. When the substitute teacher entered the class yesterday, she didn't introduce herself so Maria said, "Who are you?" It was strange that Maria had to ask ________. a. who was the substitute b. who the substitute is c. who the substitute was.
57. Please choose the correct answer from the choices presented. In literature, three main genres are distinguished: poetry, novel and drama; each of these main genres can be sub-divided into smaller genres like lyrics, sonnets and ballads for poetry. With these ideas in mind, choose the best description of the work titled “Robinson Crusoe” written by Defoe in the 18th century. a. Epic poem b. Narrative novel c. Comedy.
58. Classify following organs of speech as articulators: a. alveolar ridge b. upper teeth c. tongue.
59. Choose the correct conditional sentence that expresses the same idea as the following information I´m so busy. That’s why I don’t read bedtime stories to my little girl. a. If I was so busy, I will read bedtime stories to my little girl. b. If I were so busy, I would read bedtime stories to my little girl. c. If I weren’t so busy, I would read bedtime stories to my little girl.
60. Choose the correct option to complete the paragraph. Marin had a difficult time living in Paris. She didn't have a job to keep her busy, and she also didn't speak the language, so it was hard for her. ________, she was homesick and didn't enjoy being away from all her friends and family. a. For instance b. Furthermore c. Afterwards.
61. Read the editorial. Then use the information to complete the activities that follow. Creating a Smartphone Monster 1 I remember it as if it were yesterday—walking through the front door of my mother’s house and shouting, “It’s here!” Mom poked her head out of the kitchen, rolled her eyes when she saw the smartphone packaging, and then disappeared again. Not the best start, I’ll admit. Still, I had done my research, and all the latest studies showed that it was very helpful for seniors to learn how to use new technology devices. So, I decided I would be the one to make it happen! 2 Hours later, I finally ensured that her email account and contacts were set up. I did not relent until we reviewed a few apps and figured out the all-important ringtone. It was a long night, but even as I left, I knew that my mother had only retained half of what I had told her (if I was lucky). It’s not that there’s anything wrong with her. In fact, she’s one of the smartest people I know. It’s just that she came from a generation that didn’t have mobile phones at all, let alone smartphones. This influx of information was all very new and confusing. 3 After a few days of phone calls, her anxiety slowly diminished until my mother finally stopped asking how to do every little thing on her phone. It wasn’t until I had the urge to go shopping with her that I became conscious of the monster I had created. You see, Mom was now a smartphone addict. She used her phone for everything. Should I wonder aloud about anything at all, she would twitch excitedly and say, “They have an app for that.” She was also suddenly a compulsive email checker—the same woman who used to go days at a time between email responses. 4 But her biggest dependency was her texting. My jaw dropped when I came out of a store and found her fingers moving at the speed of light. “Mom!” I called to her, but she was “in the zone” and the temptation to continue texting was apparently too great. (I learned later that she was discussing a recipe with Aunt Lucy.) Trying to get her attention, I reached for her phone, triggering what could only be described as the mother-daughter cat fight of the century—right in the middle of a shopping mall. All the while, she clutched her smartphone as if her life depended on it, screaming “Help!” at the top of her lungs. 5 Later, that evening, I told my husband about her smartphone dysfunction and how we would have to slowly wean her away from using it. Hopefully she wouldn’t have any withdrawal symptoms. I watched him literally cry from laughing so hard. “Don’t you see the humor in all of it?” he finally asked when he saw I wasn’t smiling. I think my silence answered his question. 1. app: application, or program for a mobile device. According to the information, which of the following statements is false? a. The author believes her mother should not use her smartphone as much as she does. b. The author’s mother does not want the smartphone when the author gives it to her. c. The author’s mother never really understands how her smartphone works.
62. Elija la opción correcta para completar la conversación. R: Son casi las 8 pm y John aún no está aquí. ¿Dónde está el? B: No lo sé. Él ________ a las 7 pm. a. se suponía que había llegado b. mejor llego c. debe haber llegado.
63. Read the editorial. Then use the information to complete the activities that follow. Creating a Smartphone Monster. 1 I remember it as if it were yesterday—walking through the front door of my mother’s house and shouting, “It’s here!” Mom poked her head out of the kitchen, rolled her eyes when she saw the smartphone packaging, and then disappeared again. Not the best start, I’ll admit. Still, I had done my research, and all the latest studies showed that it was very helpful for seniors to learn how to use new technology devices. So, I decided I would be the one to make it happen! 2 Hours later, I finally ensured that her email account and contacts were set up. I did not relent until we reviewed a few apps and figured out the all-important ringtone. It was a long night, but even as I left, I knew that my mother had only retained half of what I had told her (if I was lucky). It’s not that there’s anything wrong with her. In fact, she’s one of the smartest people I know. It’s just that she came from a generation that didn’t have mobile phones at all, let alone smartphones. This influx of information was all very new and confusing. 3 After a few days of phone calls, her anxiety slowly diminished until my mother finally stopped asking how to do every little thing on her phone. It wasn’t until I had the urge to go shopping with her that I became conscious of the monster I had created. You see, Mom was now a smartphone addict. She used her phone for everything. Should I wonder aloud about anything at all, she would twitch excitedly and say, “They have an app for that.” She was also suddenly a compulsive email checker—the same woman who used to go days at a time between email responses. 4 But her biggest dependency was her texting. My jaw dropped when I came out of a store and found her fingers moving at the speed of light. “Mom!” I called to her, but she was “in the zone” and the temptation to continue texting was apparently too great. (I learned later that she was discussing a recipe with Aunt Lucy.) Trying to get her attention, I reached for her phone, triggering what could only be described as the mother-daughter cat fight of the century—right in the middle of a shopping mall. All the while, she clutched her smartphone as if her life depended on it, screaming “Help!” at the top of her lungs. 5 Later, that evening, I told my husband about her smartphone dysfunction and how we would have to slowly wean her away from using it. Hopefully she wouldn’t have any withdrawal symptoms. I watched him literally cry from laughing so hard. “Don’t you see the humor in all of it?” he finally asked when he saw I wasn’t smiling. I think my silence answered his question. 1 app: application, or program for a mobile device. The author refers to the latest research studies in paragraph 1 to support her opinion that ____. a. her mother uses a smartphone too much b. new technology is developed all the time c. her mother should have a smartphone.
64. In imperative sentences, as in the examples: Come here! Close the door! Listen to me! the intonation is: a. Rising b. Neutral c. Falling.
65. Which of the following is true about universal grammar? a. It is an explanation of knowledge and learning that is based on the assumption that all learning is first social then individual. b. It is considered as innatist linguistic knowledge and argues that some grammar rules are hard-wired into the brain. c. It is an approach that emphasizes how the human mind receives, process, stores and retrieves information in learning and retrieving information.
66. Complete: ____ is concerned with practical issues of language and communication. It deals with language acquisition, cognition and teaching methods; language testing and assessment. a. Language teaching b. Applied Linguistics c. Theoretical models of language and language use.
67. Choose the letter of the answer that best completes the following information. In order to show cause and effect, it is necessary to use two comparative adjectives; for example: a. The more enjoyable the meal, the more satisfied the customers. b. The Pizza Palace is just as old as Joe’s Pizzeria. c. A slice of pizza is getting more and more expensive.
68. Which statement best represents the idea of a ¨Culture¨? a. The art and music produced by a group of people b. The growth or cultivation of the minds of humans, in this moment and over time c. The written words produced by a group of people.
69. Choose the best answer from the choices. “universal grammar” theory implies that: _____________________. a. all humans create their own set of grammar rules to be able to communicate properly. b. there is a connection between speech and human relations and human understandings of the world. c. the ability to learn grammar is built into the human brain from birth regardless of language.
70. Lea y elija la definición correcta de motivación integradora. a. Se basa en el deseo de saber más sobre la cultura y la comunidad del grupo de idiomas de destino e incluso en el deseo de ser más como miembros de ese grupo. b. Consiste en un conjunto de categorías predeterminadas que se utilizan para registrar y describir comportamientos de enseñanza y aprendizaje. c. Es esencialmente práctico, como la necesidad de aprender el idioma para obtener un mejor trabajo. .
71. Read the information below and complete the sentence. Clarification requests, Elicitation and Repetition are some techniques that teachers use as… a. Corrective feedback. b. Observation schemes. c. Instructional approach.
72. Read the information below and choose the correct option. Nelly Ellis (2001) and others researchers have hypothesized that one of the most important variables in predicting success for learners in many language situations is the active processing of information. It is called: a. Pimsleur Language Aptitude Battery (PLAB) b. Modern Language Aptitude Test (MLAT) c. Working Memory (WM).
73. Choose the option that contains the three errors in the following paragraph? At the moment, all five babies are sleeping and Mary wants to sleep too. All five babies sleep rarely at the same time. There usually is at least one baby up or crying. Mary looks in their room to make sure they are all OK. They are all sleeping, and they are looking beautiful. a. are sleeping / sleep / look b. sleep rarely / looks / are all sleeping c. sleep rarely / There usually is / are looking.
74. Applied Linguistics is usually defined by linguists as the academic discipline that is concerned with the relation of knowledge about language in general to decision making in the real world, and it is because it: a. expands the knowledge about the world languages b. offers solutions to language – related problems c. represents one of the requirements for students previous to become efficient teachers.
75. Which of the following is correct: a. Literature is confined to historical documentation of a group of people b. Language, Literature and Culture are completely different aspects of Human existence c. Language and Literature are used to express and share Culture.
76. Choose the correct option for each sentence. a. Sharon will have to retake the course before she can get her degree b. Sharon is going to has to retake the course before she can get her degree. c. Sharon will has to retake the course before she can get her degree .
77. Read the next pairs of sentences. Which sentence pair can be joined with “even though”? a. The scientist visited Antarctica. He had to leave his family. b. The scientist was careful. Her work was important. c. Scientists want to visit Antarctica. There are many things to study.
78. Why may Sociolinguistics be interesting for educators? Choose the option that does not answer the previous question. a. Sociolinguistics may help understand the development and changes of Languages b. Because Standard English belongs exclusively to certain English speaking countries. c. Teachers must know the fact that there is dialect variation in the target language just as there is in their native language. All language teaching implies the best dialect (standard) to teach. .
79. In the clause “It is a beautiful day to take a walk”, “ful” modifies the noun “beauty”, changing its word category type. This is an example of how ______work. a. inflectional suffixes b. derivational prefixes c. derivational suffixes .
80. Read the outline of a cause-and-effect essay. Then answer the questions. 1. Introductory Paragraph Thesis statement: “Starting a blog about a topic of interest can have many benefits, including organizing personal ideas and information, making friends who share the same interest, and learning and sharing new information with others over time.” 2. Body paragraph 1: organizing personal ideas and information example: gardening 3. Body paragraph 2: making friends who share the same interest meeting people through comments 4. Body paragraph 3: learning and sharing new information with others over time solving problems through blog discussions 5. Concluding Paragraph What is the essay about? a. the benefits of gardening b. the benefits of searching the Internet c. the benefits of having a blog.
81. Read the paragraph. Then answer the questions that follow. Gilles Tréhin is an interesting savant who created an imaginary city in his head. Gilles has drawn hundreds of sketches to provide the details of this city, which he named “Urville.” He has been able to draw three-dimensional objects since he was five years old. He has also written history of Urville and published a book about the city. Gilles is an example of a savant who lives in his own world. What is the topic sentence of the paragraph? a. Gilles Tréhin is an interesting savant who created an imaginary city in his head. b. Gilles has drawn hundreds of sketches to provide the details ofthis city, which he named “Urville.” c. He has been able to draw three-dimensional objects since he was five years old.
82. According to some grammarians, there are three types of ____: Simple, Compound, and Complex a. clauses b. functions of grammar c. sentences.
83. Choose the option that contains the correct indirect form of the sentence below. Mary said, “I do not have any money now.” a. Mary said that she did not have any money now. b. Mary said that she has not had any money that day. c. Mary said that she did not have any money then. .
84. According the Contrastive Analysis Hypothesis (CAH), which of the following may be one of the causes of learners’ errors? a. Students transfer knowledge their first language to their new language. b. Student´s did not practice the correct forms enough to make the habit c. Students did not pay close enough attention to the languages structures they were taught. .
85. Which of the following is correct: a. Literature is confined to historical documentation of a group of people b. Language and Literature are used to express and share Culture c. Language, Literature and Culture are completely different aspects of Human existence .
86. Local dialects or accents are often useful in literature because: a. It is more romantic to hear someone speak with an accent b. It allows the reader to understand sarcasm and humor c. It allows the reader to know something about the person without having to state it.
87. Elija la aplicación más adecuada de Sociolingüística para la educación: a. Proporciona una descripción de las variaciones del lenguaje y explica los cambios del lenguaje a través del tiempo. b. Explica el proceso de adquisición del lenguaje y los problemas relacionados con el bi y el multilingüismo. c. Se centra en el papel del comportamiento del lenguaje para la educación y la adquisición del lenguaje.
88. Read the context of the paragraph and choose the correct option to complete it. I can’t believe you forgot ___________ the proposal for the conference. Now we can’t present the results of our study until the next year! You should’ve paid better attention to the deadlines! a. submitting b. submit c. to submit .
89. The clause “Anabelle is laying on her bed” belongs to the type of copula verb called: a. Ascriptive b. Locative c. Equative.
90. By June, I’ll have been working for seven months. What tense is the example above in? a. Future progressive. b. Future Perfect c. Future perfect progressive.
91. Subcategorization can be different for each verb. For instance, the example “Marcia went to the bakery” belongs to the subcategorization rule in which… a. Verbs are followed by a motion phrase. b. Verbs are followed by present-participial verb phrases. c. Verbs are followed by a noun phrase and an adjective phrase. .
92. Choose the option that does NOT fit the category of language–related problems: a. scientific description and classifications of world languages b. translation, interpretations, language usage in general, literary genres and styles, machine translation c. use and analysis of language variations, dialects, registers, and language codes.
93. Read the text about “Love of Language”, answer the following question. Choose the correct option. LOVE OF LANGUAGE 1 Speaking as an American, I'm glad that English is a global language. This means I can usually talk to people in many places in Asia, the Middle East, Europe, and Africa. At the same time, I sometimes feel embarrassed when I compare myself to people who know two or three languages. I feel kind of inferior compared to them. At least I've learned some different dialects of English and I understand how English is used in different parts of the world. 2 I'd still like to learn at least one other language—Japanese. Next year, I'm planning to teach English in Japan, and I want to be able to make friends with the people there. I know that a lot of Japanese speak English fluently, but I definitely want to learn about their culture, including their slang and maybe a little profanity! 3 I also want to understand Japanese proverbs more deeply. One famous proverb goes something like this. Fall seven times, stand up eight. I'd like to hear that proverb in Japanese and talk about its meaning with a native speaker. Talking with a Japanese person about it will be much more interesting than reading about proverbs in a book. 1. Which sentence best describes the writer's attitude toward English and other languages? a. Knowing other languages is more important than knowing English is. b. It is convenient to speak English but also important to know other languages. c. English speakers don't really need to learn other languages.
94. Read the following description, then match it to the corresponding hypothesis. It is a metaphorical barrier that prevents language acquisition a. The Affective Filter hypothesis b. The Acquisition-Learning hypothesis c. The Natural Order hypothesis.
95. Complete the following statement: In phonological analysis, segmental features refer to which correspond fairly well to vowel and consonant sounds. a. morphemes b. clusters c. phonemes.
96. Read The information below and choose the correct option. The error “She is a lady beautiful” made by Spanish speakers who is learning English is an example of: a. Interference error b. Instruction error c. Processing error.
97. The supra-segmental aspects of speech are represented by: a. consonants and vowels b. single vowels, voiced and voiceless consonants c. stress, length, pitch, tone.
98. What is the meaning of the following sentence? By the end of this month, our company will have been doing computer repairs for three years. a. The team started their business almost three years ago. b. The team will continue offering computer services for three more years. c. The team will start doing new computer services.
99.. Choose the words that present vowel reduction phenomenon: a. bottle, window, sky, cake b. bag, tone, pitch, star c. nation, certain, student, pencil.
100. Choose the option that matches with the following statements: “English has both regular and irregular verb forms, together with regular and irregular noun plural forms”, “English language has formal and informal greeting forms”, “Reduced forms are present in informal spoken language”. a. Register b. Prescriptive approach to language study c. Descriptive approach to language study.
101. Read the entire story. Use the information to choose the correct answers. Tim Stark was a busy and successful management consultant in New York City. He was under a lot of stress from his job. His back hurt. To relax on weekends, Tim started growing tomatoes in his apartment. First he grew a few tomatoes. Then he grew more. A year later, his apartment was full of 3,000 healthy red tomato plants! Then, one day, Tim made a decision that needed a lot of courage. He quit his management consultant job to become a farmer. Now Tim raises a variety of crops on a small farm in Pennsylvania, two hours away from New York. He is still under a lot of stress: he often works until sunset, and has a lot of responsibilities, such as watering his crops, harvesting them, and bringing them to the market in his old white truck. Tim says his back still hurts, but he likes living close to nature and working in his fields. Tim believes being a farmer has many advantages. “My tomatoes need me, and I need them,” he says. Tim’s favorite part of being a farmer is probably . a. working until sunset b. driving his truck c. walking in his fields.
102. Choose the option that best completes the following statement: _____ claim that children are born with a “sort of template” which enables them to acquire language a. Sophists b. Behaviorists c. Structuralists.
103. Aspiration refers to: a. Audible escape of air following some consonant sounds b. activity of the vocal cords c. movement of tongue inside the vocal tract.
104. According the Contrastive Analysis Hypothesis (CAH), which of the following may be one of the causes of learners’ errors? a. Students transferred knowledge their first language to their new language. b. Student´s did not practice the correct forms enough to make them a habit c. Students did not pay close enough attention to the languages structures they were taught.
105. For the purposes of Culture and literature, this concept implies the possible application of interpreting by means of performance as well as analysis. a. Answering b. Interpretation c. Historical context.
106. Read the statement and choose the correct option. One of the learners characteristics in the process of learning a foreign or second language is personality. In fact, the predisposition of learners toward or away from communicating in a second/foreign language involve social, individual, situational, and motivational factors. It refers to________________________________. a. Modified interaction b. Willingness to communicate c. Cognitive maturity.
107. Which of the following sentences is correct? a. We will have completed already our research by Monday. b. We will not have taken yet the final exam. c. We’ll already have finished by then. .
108. What are tag questions used for? a. To reject information. b. To check information. c. To protect information.
109. COMMUNICATIVE GRAMMAR Choose the best answer for the following sentence. What should be _______ about the student lounge? a. do b. did c. done.
110. CULTURE AND LITERATURE Which of the types of questions are best to ask when analyzing a text? a. Who, what, where, when, why, how, what if b. Why was this story written? c. Is this story interesting and would I recommend it to another person?.
111. Read the text about “Love of Language”, answer the following question. Choose the correct option. LOVE OF LANGUAGE 1 Speaking as an American, I'm glad that English is a global language. This means I can usually talk to people in many places in Asia, the Middle East, Europe, and Africa. At the same time, I sometimes feel embarrassed when I compare myself to people who know two or three languages. I feel kind of inferior compared to them. At least I've learned some different dialects of English and I understand how English is used in different parts of the world. 2 I'd still like to learn at least one other language—Japanese. Next year, I'm planning to teach English in Japan, and I want to be able to make friends with the people there. I know that a lot of Japanese speak English fluently, but I definitely want to learn about their culture, including their slang and maybe a little profanity! 3 I also want to understand Japanese proverbs more deeply. One famous proverb goes something like this. Fall seven times, stand up eight. I'd like to hear that proverb in Japanese and talk about its meaning with a native speaker. Talking with a Japanese person about it will be much more interesting than reading about proverbs in a book. Which example best supports the opinion that knowing a language is the key to knowing a culture? a. The writer wants to learn Japanese to understand people, proverbs, and slang. b. The writer learned different dialects of English while traveling around the world. c. The writer feels inferior to people who know several languages and cultures.
112. Choose one of the alternatives that best answers the question. In a study of grammatical morpheme acquisition, it was shown that the use of morphemes acquired at a later date, indicates earlier morphemes have been mastered, but the reverse is not true. This suggests which of the following? a. There is a developmental sequence in first language morpheme acquisition. b. Morphemes whose sounds are harder to perceive or which are difficult to pronounce are learned later. c. Children learn the easiest morphemes first and then progress to more difficult ones.
113. COMMUNICATIVE GRAMMAR Complete the following sentence: It might __________ fixed by the summer. a. be b. is c. being.
114. When student has difficulties with language behavior, it can be manifested through: a. Uncontrolled code switching or incorrect use of grammatical rules b. Lack of motivation for language learning and disruptive behavior during the class c. Problems with language and speech development and production.
115. What is the meaning of the following verb: turn down ? a. move b. reject c. try.
116. What is the meaning of the following sentence? My parents ought to have move away from that neighborhood. a. They’re are going to move, but they’re not sure when. b. Moving was a good idea, but they didn’t do it. c. There are moving away next week.
117. It refers to the area of understanding where he or she can move to the next level with help from another person. a. Zone of Proximal Development b. Information Processing Model c. The Input hypothesis.
118. Choose the correct transition to connect the ideas for the next sentence. Sentence: The amount of polar ice has been decreasing. _______ , the polar bear has lost part of its natural environment. a. Therefore b. Because c. Since.
119. What is the meaning of the following sentence? They got me to do research on tropical birds. a. I agreed to do the research. b. I didn’t agree to do the research. c. They forced me to do the research.
120. The following excerpt contains two errors. Choose the option that has the same excerpt written correctly. The new album by the band The White Bears had gone on sale last month, and it had selling poorly in the first two weeks until the band is appearing on The Nightly Show. a. The new album by the band The White Bears had gone on sale last month, and it had been selling poorly in the first two weeks until the band appeared on The Nightly Show. b. The new album by the band The White Bears is going on sale last month, and it had selling poorly in the first two weeks until the band appeared on The Nightly Show. c. The new album by the band The White Bears had gone on sale last month, and it had selling poorly in the first two weeks until the band is appearing on The Nightly .
121. CULTURE AND LITERATUIRE Which of the following is not a type of Literature? a. Twitter b. Oral Story telling c. Talking on a cell phone.
122.CULTURE AND LITERATURE Literature is an important part of any Educational System because: a. It´s often funny or entertaining b. It conveys the beliefs, behaviors and ideas of a cultural group c. It often shows the errors of a cultural group.
123. Local dialects or accents are often useful in literature because: a. It allows the reader to understand sarcasm and humor b. It allows the reader to know something about the person without having to state it c. It is more romantic to hear someone speak with an accent.
124. There are various types of non-finite clauses which are distinguished mainly by what type of finite clause they modify. Thus, the sentence “The man reading the newspaper didn’t hear my song” illustrates the following structure: a. Reduced adverbial nonfinite clause. b. Reduced relative nonfinite clause. c. Free participle nonfinite clause.
125. Decide the best answer to the following statement: The whole time she was talking, I_____________about asking her on a date. a. think b. thought c. was thinking.
126. Read and choose the correct definiton of Classroom Observation Scheme. a. It consists of a set of predetermined categories used to record and describe teaching and learning behaviours b. It is essentially practical, such as need to learn the language in order to get a better job. c. It is based on the desire to know more about the culture and community of the target language group and even a desire to be more like members of that group.
127. Complete the following statement: The articulatory properties of vowels differ in several aspects from those of consonants, mainly in the degree of vocal tract opening. It refes to: a. Acoustic difference b. Suprasegmental aspects c. Physiological difference.
128. Read the history. Choose the correct option to complete it. I worked in an international office whit co-worker who speak several different dialects of English. Sometimes we teach each other different 1. _________ expressions that are used like Toronto, Canada, and Sydney, Australia. Most of co-workers are quite talkative, so we have very interesting conversations. However, one co-worker is a/an 2. ________ He doesn’t like to talk much, but hi is quite assertive. When some people start to use 3. ______, he tells them to stop. a. 1. Profanity 2. exception. 3 slang b. 1. slang 2. profanity. 3 exception. c. 1. Slang 2. Exception 3. Profanity.
129. COMMUNICATIVE GRAMMAR Complete the following sentence. Mr. Smith, …………. dog I walk lives next door. a. whose b. who c. whom.
130. What is the meaning of the following sentence? I shouldn’t have called him. a. I didn’t call him, and now I regret it. b. I called him, and now regret it. c. I called him, and now I ‘m happy .
131. COMMUNICATIVE GRAMMAR Complete the following sentence: It might _______fixed by the summer. a. be b. being c. is.
132. COMMUNICATIVE GRAMMAR When I left the museum, she_________ me her phone number. a. was giving b. give c. gave.
133. COMMUNICATIVE GRAMMAR What tense is the example below in? By June, I’ll have been working for several months a. Future perfect b. Future perfect progressive c. Future progressive.
134. Complete the following statement: The articulatory properties of vowels differ in several aspects from those of consonants, mainly in the degree of vocal tract opening. It refes to: a. Acoustic difference b. Suprasegmental aspects c. Physiological difference.
135. Choose the correct option to complete the conversation. A: It's almost 8 p.m. and John isn't here yet. Where is he? B: I don't know. He _____ by 7 p.m. a. had better arrived b. was supposed to have arrived c. must have arrived.
136. Match each word on the left to a word with the same meaning on the right. weep smelly awful understand.
137. Semantic units of language in use include Text and Discourse, both are fundamental in coherent communication, which means that they… a. are used to form correct grammatical structures. b. are semantically and pragmatically coherent in real communication. c. are specific structures used to express a single message.
138. Words containing rounded vowels are: a. garage, may, pain, leave, even, save b. auction, over, boat, octopus, though c. above, lack, pine plough, bus .
139. Which are the five basic active articulators? a. The lip, The flexible front of the tongue, The middle/back of the tongue, The root of the tongue together with the epiglottis and The larynx b. The lip, The flexible front of the tongue, The throat, The root of the tongue together with the epiglottis and The nose c. The lips, The tongue, The soft palate, The epiglottis and The larynx.
140. Literature is an important part of any Educational System because: a. It´s often funny or entertaining b. It conveys the beliefs, behaviors and ideas of a cultural group c. It often shows the errors of a cultural group.
141. I f you use Have+ objective + base form of the verb. a. To persuade a person or animal to do something b. To force a person or animal to do something c. To cause a person or animal to do something. .
142. I identify the concluding sentence that best paraphrases Compare to many animals. Our own sense are almost………. a. human’s senses are better than animals b. cats are more sensitive than dogs c. Animals ‘senses are better than humans’are .
143. Read the following paragraph A: Human activities, including pollution……. B: Many species of plants and animals C: Contact your local wildlife …… a. B, A, C b. C, B, A c. A, C, B.
144. Read the concepts below and match with the corresponding word It discourages the risk- taking which is necessary for progress in language learning. This is often considered a particular problem for adolescents who are more self- conscious than younger learners are. It argues that a person is well suitable to language learning. Feelings of worry and stress that many students experience when earning a second language.
SECOND TERM: 145. Reorganize the given outline (from 1 to 6) so it meets the right organization of a ‘process’ paragraph. 1. When the popping has finished, take the pot out of the stove, spread salt to taste and enjoy it. 2. As soon as it starts popping, keep moving the pot without uncovering it 3. As it was seen, making popcorn is a matter of four easy steps. 4. This process starts by heating oil in a pot. 5. The process to make popcorn is made up of four simple steps. 6. Then, spread the kernels in the pot and cover it. a. 4 - 6 - 5 - 2 - 3 – 1 b. 4 - 6 - 2 -1 - 3 – 5 c. 5 - 4 - 6 - 2 -1 – 3.
146. Which of the following can be used to check the student’s skill at word recognition? a. Reading aloud b. Reading logs c. Speed reading graphs.
147. Choose the option that best completes the following statement: __________ is a completed study that reports an investigation or exploration of a problem, identifies questions to be addressed, includes the collection and analysis of data, and advances an interpretation of the data. a. Investigation b. Portafolio c. Research report.
148. From the following examples, which one does not refer to induction? a. I have a bag of many coins, and I’ve pulled 10 at random and they’ve all been pennies, therefore this is probably a bag full of pennies. b. Every time you eat peanuts, your throat swells up and you can’t breathe. This is a symptom of people who are allergic to peanuts. So, you are allergic to peanuts. c. Since all squares are rectangles, and all rectangles have four sides, so all squares have four sides.
149. One of the advantages of a ___________ used for sequencing content is that it provides easily monitored recycling of material. a. Linear model b. spiral model c. Matrix model.
150. If a test contains natural language, meaningful and relevant topics and contextualized items that replicate real- world tasks, we say that this test is _________. a. valid b. reliable c. authentic.
151. Curriculum implies all the issues related_____________, which are conceived as a coherent whole with a specified purpose. a. to teaching, monitoring and assessing English teaching b. to planning, implementation and evaluation of a series language learning events c. to educational policies and laws for English teaching in schools and high schools .
152. Choose the option that contains one of the steps to implement Learning Centers in the classroom? a. Identifying skills to be practiced b. Flashing light signals c. Vocabulary instructions.
153. A device used to evaluate open-ended oral and written responses of learners is called a. Grading b. Scoring c. Rubrics.
154. Which of the following steps DOES NOT correspond to the strategy Collecting and Processing words? a. Introduce synonyms and add them to the word collection with the original word. b. Identify unfamiliar words in reading sections c. Identify key vocabulary.
155. Read the example below and identify the option that contains the strategy that is applied by the teacher: In an English course the students wait excitedly the end of a unit because they know that at that time -after finishing the unit- the teacher takes them outside the classroom to practice what they have learnt. When that moment comes, before going out, the students take their notebook and follow the teacher to the place he/she has chosen to go. a. The teacher uses the activates background knowledge strategy to relate previous knowledge to the new class. b. The teacher applies the small groups strategy and give clear instructions to involve all learners in the activity. c. The teacher has established the predictable routines and signals strategy in which students react without any detailed guidance.
156. This strategy gives teachers the opportunity to observe students as they read from texts at their instructional reading levels. a. Read, Pair, Share b. Read-Aloud plus c. Guided reading.
157. ____________measure learners’ ability within a classroom lesson, unit, or even total curriculum. a. Proficiency tests b. Achievement tests c. Diagnostic tests.
158. It involves everyone in the group, people can speak directly to each other and it creates equality through the groups; these are some advantages of: a. Circle arrangement b. Horseshow arrangement c. Separate tables arrangement.
159. Choose the option that best completes the following statement: The______ is the subject matter of the study, the________ sets for the intent of the study, and the_______ will be answered by the researcher based on data collected in the study. a. Research problem/ topic / research questions b. Investigation/ topic / instruments c. Topic/ purpose statement/ research questions.
160. During______, teachers monitor student reading processes and check that texts are within students’ grasps, allowing students to assemble their newly acquired skills into a smooth, integrated reading system. a. Read, Pair, Share b. Guided reading c. Repetition and innovation.
161. Select the activity that helps students work on intensive reading. a. Work in groups of three students and share information about one of the books you read the last week. Then prepare an oral book report for your classmates. b. Read the student’s introduction to an essay. Then answer the following questions: • Does the writer spark the reader’s interest at the very beginning? • Does the writer go from broad general statements to specific statements? c. Using the information that you have read about cause-and-effect essays, write a well-written essay focused on the positive or negative effects of social media.
162. Choose the option that best fits the definition below. __________is designed to develop basic communication skills. The development stages are: (1) Comprehension (preproduction), (2) Early Production, and (3) Speech Emergence. a. Natural approach b. Language Experience Approach c. Total Physical Response.
163. Select the questions that you will use if you are focusing on needs about genres and discourse types as well as sociolinguistic skills. a. Are the learners happy with the course?/ What content matter will the learner be working with? b. What activities will I get the learners to do today?/ Should learners be doing homework? c. What will the language be used to do? / What language uses is the learner already familiar with?.
164. Choose the answer that best completes the following sentence: A reliable test ___________ a. measures what it proposes to measure. It relies as much as possible on performance. b. demonstrates the degree of correspondence of the characteristics of a given language test task to the features of a target language task. c. is consistent and dependable. If the same test is given to the same student or matched students in two different occasions the test should yield similar results.
165. In the context of curriculum design, there are numerous factors that have one or more effects. For example, the factor “the large size of the class” could have the following effects on the curriculum design. a. Need to use special large class techniques like oral reproduction, blackboard reproduction, pair and group work. b. Need to adapt content and teaching material to the level of the students, that is, to consider students’ needs and wants. c. Need to hire specialized and experienced facilitators on teaching English for specific purposes.
166. Which of the following are types of reading assessment: a. Form-completition, Cloze selection, and Grammatical transformation. b. Perceptive, Interactive, and Extensive. c. Imitative and Intensive.
167. Read the following research question and choose the research design it belongs to: How well do homeschoolers who attend college do compared to the ones who attended traditional schools? a. Quantitative research design b. Qualitative research design c. Mixed research design.
168. One of the principles of language teaching, in curriculum design, refers to language system. This principle is applied when the teacher ______________. a. focuses attention on predominantly regular features of the language in order for learners to be able to use the language creatively. b. checks the sequence of items to see that strongly related items are not presented together to avoid learners’ confusion. c. designs activities where the learners work cooperatively to learn the content of the language.
169. Read the following information and then choose the correct response. Informal assessment can take a number of forms. These forms can be; for example, _____. a. Say to your students expressions like “Good work”; “Did you say can or can’t?”; “I think you meant to say you broke the glass, not break the glass”. Another example is when teachers design tasks to elicit performance without recording results. b. A proficiency test that tests global competence in a language and that is not limited to any one course, curriculum, or single skill in the language; rather, it tests overall ability is a form of informal assessment. c. Diagnose aspects of a language that a student needs to develop or that acourse should include .
170. _________ is a powerful application of context-embedded academic language that serves to support English learners in successful classrooms participation. a. Total physical response b. Modeled talk c. Vocabulary role-play .
171. Decide which of the following statements correctly works as the general statement of an introductory paragraph. a. Data suggests that the industrialized nations are in fact making considerable progress in reducing their levels of pollution and other environmental damage; however, developing countries such as China and India are industrializing rapidly, and it is widely believed that this industrialization is likely to have significantly negative environmental effects. b. The need to limit human impact on the environment and to avert catastrophic environmental change is unquestionably the most significant long-term challenge facing policymakers today. c. This essay will argue that it is possible for today's developing countries to industrialize without serious environmental compromises, but only if developed countries are willing to provide financial and political assistance. .
172. A teacher should know that to teach a beginner group, and to be sure that the input can be understand through…… a. The activities where all the language items are with their previous experience. b. The use of visual aids and contextual support for new language including pictures, gestures, mime, objects, and experiences out of class. c. Simple group activities where learners listen.
173. Decide whether these statements are examples of introductory paragraphs, body paragraphs, or concluding paragraphs. 1. The main reason for people being unhappy with censorship in any form comes from their mistaken belief that an individual's private actions have no effect on others. 2. A related reason for dissatisfaction with censorship appears to be a false notion of the idea of freedom of expression of ideas. a. Body paragraphs b. Introductory paragraphs c. Concluding paragraphs.
174. Why are aisles between the rows important for the teacher? a. To teach grammar lessons properly. b. To monitor students’ progress. c. To have a clear view of all the students.
175. Questions such as: What are the students´ current language proficiency?, How much input do teachers have on curriculum, syllabus and material selection and use?, and What are the physical environments in which the course will be delivered? are applied to gather information about the that influence on the curriculum design. a. factors concerning students, teachers, and teaching and learning situations b. process of curriculum like needs analysis, monitoring and assessing c. aspects concerning course participants, principles, and context.
176. In curriculum design, there are many topics that are addressed in a number of different places within the program requirements while other topics are addressed by specific courses. In order to integrate those topics at a meso and microcurricular level, curriculum designers use _______________as a paradigm for formalizing the integration across the curriculum. a. environment analysis b. academic threads c. learning outcomes.
177. Materials need to have _________ which suit the students and their needs since some of them are appropriate for a particular class but not for other. a. an appropriate and consistent design b. a textbook, a workbook, pictures, CDs, videos, handouts, white board among others c. an underlying instructional philosophy, approach, method and technique.
178. If an EFL course contains a mixture of high- and low-frequency items that does not give the best available return for learning effort, the best way to overcome this problem is by ________________. a. including all the high-frequency items which are at the appropriate level for the learners. b. including activities and strategies that allow learners to practice an obvious learning goal. c. deciding to reduce teacher talking time and hence students’ communication in class. .
179. ACADEMIC WRITING Read the following paragraph. Then choose the option that indicates the type of essay used. There were several reasons behind our decision to move to Flemington. The first occurred about 18 months ago when Mark and I decided to start a family. We were living in a one-bedroom apartment and we knew that we wanted to move into larger quarters before we had a baby. We began to look at houses. Then, much sooner than expected, I got pregnant. Soon after that, Mark’s company announced that they were relocating to Flemington, which was in a less expensive part of the state, about 90 miles south of us. Mark’s company had been good to him, and they were one of the few around with excellent benefits, family friendly policies, and a child-care center on site. With a baby on the way, these things were imperative for us. Since I ran my graphic arts business from home, I wasn’t bound to any particular place, so we began looking at real estate in Flemington and also did some research on their school system as well as the overall community. We were very excited about what we found— reasonable housing costs, great schools, and a lively town. Mark then accepted the relocation offer and we found a beautiful old Tudor house. We’ll be moving about a month before the baby is due. Let’s hope she doesn’t decide to come early. a. compare and contrast essay b. cause and effect c. opinion.
180. Its purpose is more notetaking than record-keeping. According to the statement we can point out to: a. An interview b. An observation c. A questionnaire.
181. Select the item that states the aspects that English teachers must consider when selecting authentic reading material for a course. a. The needs of the community, the educational policies and the teaching resources. b. Suitability of content, exploitability, readability and presentation. c. Approach, the financial cots, the location of the educational institutions.
182. Read the following statement and choose the information that There are some measures of central tendency; one of them is the mode, which is. a. the total of the scores divided by the number of scores. b. the middle score among all scores. c. the score that appears most frequently in a list of scores.
183. What is the correct order of the stages of language development? a. Preproduction characteristics. Early production characteristics. Speech emergence characteristics. Intermediate fluency characteristics. b. Early production characteristics. Speech emergence characteristics. Preproduction characteristics. Intermediate fluency characteristics. c. Speech emergence characteristics. Preproduction characteristics. Early production characteristics. Intermediate fluency characteristics.
184. Needs analysis can play a major role in determining the _____________ a. number of students for the course. b. steps of the environment analysis. c. content of courses, particularly for language items.
185. Within the Extensive Reading tasks, a teacher may ask the students to _________. a. find and circle/underline all irregular verbs b. write a short summary of each paragraph of reading c. find all phrasal verbs used in a text .
186. Analyze the following case: To prepare her second graders for a field trip to the zoo, Ms. Allen brings in photographs of zoo animals. She introduces the animals to the students using the photographs and creates a chart by displaying the photographs and listing key information about each animal, such as the names used for the baby animals, and the animal’s natural habitat. A map of the zoo is displayed in the classroom and Ms. Allen teaches a lesson on asking questions and gathering information during the zoo trip. The class brainstorms questions they will ask the zoo guides about the animals and the students practice taking notes on their information sheets. What strategy to encourage active involvement is Ms. Allen using in her class? a. Visual scaffolding b. KWL and Data Charts c. Preview / Review.
187. Formal speaking involves taking long turns. This planning can be done in several ways. Choose the correct one: a. The speaker should pay attention to turning ideas into speech while they Speak. b. The learners should have the opportunity to work through a series of spoken tasks. c. The speaker can look at the ideas that will be presented and find an effective way of.
188. Which one of the following options corresponds to informal assessment? Informal assessment _________________ a. involves evaluation through the use of final exams in a course that aim to measure or summarize what a student has grasped. b. is virtually always nonjudgmental, in that you as a teacher are not making ultimate decisions about the student’s performance; you are simply trying to be a good coach. c. implies the use of tests that are usually time-constrained and draw on a limited sample of behavior.
189. This term is intended to denote a context in which a teacher interviews a student for a designated assessment purpose. We call it as: a. An interview b. A conversation c. A dialogue.
190. One of the specific characteristics of Qualitative Research is that _____________. a. we need to develop a deep understanding of the central phenomenon. b. we will include a substantial amount of information in the Literature Review. c. we analyze trends, compare groups, or relate variables using statistical analysis.
191. Analyze the following case: In Mr. Pollitt’s class, the students have some confusion understanding the word order of some grammatical structures. In order to help them, Mr. Pollitt has taken some sentences from collaborative retell and cut apart each word; then he has his students put sentence back together like a puzzle. After that, he asks to create new and more complex sentences as the students progress.What strategy for the development of language is Mr. Pollitt using? a. Sorting activities b. Syntax surgery c. Collecting and processing words.
192. The results of a needs analysis will generally consist of information taken from several different sources and summarized in the form of ranked list of different kinds like: a. Lists of principles for organizing and sequencing language content. b. Lists of perceived difficulties with different aspects of linguistic use. c. Lists of the most frequent words and expressions learners need to know.
193. Read the following example, then decide which type of assessment it refers to: During an English unit, the teacher administers a weekly quiz that addresses all of the material covered for the week. The quizzes are supposed to motivate students to study for the summative unit as well as provide students with a sample of the types of questions they may encounter on the unit test. a. Formative assessment b. Informal assessment c. Summative assessment.
194. This seating arrangement favors students’ interaction, students engage in the roles of speaker and listener. It provides reassurance for expressing ideas. a. Orderly rows b. Horseshoe c. Circle.
195. Choose the option that best matches this definition: Its purpose to be as useful as possible to the people who are going to use it. There are different formats; in most of the cases, the educational institutions are the ones who provide teachers with them, if not you can use the one you prefer. a. Pair work or group work b. The value of repetition c. Lesson plan.
196. The steps to be followed by the teacher to apply the Cooperative Learning strategy are: ____________________ a. 1. Intervene to ensure full participation; 2) give each team a task to complete and remind each member of the roles they are expected to serve to assist the others in completing the task; 3) provide an opportunity for the groups to report back to the class at the end of the assigned time. 4) give the team members cards that identify their assigned roles and list clear description on their duties; 5) Provide a team-building activity as warm-up. b. 1) Provide an opportunity for the groups to report back to the class at the end of the assigned time; 2) give the team members cards that identify their assigned roles and list clear description on their duties; 3) Intervene to ensure full participation; 4) Provide a team-building activity as warm-up; 5) give each team a task to complete and remind each member of the roles they are expected to serve to assist the others in completing the task. c. 1) Provide a team-building activity as warm-up; 2) Give the team members cards that identify their assigned roles and list clear description on their duties; 3) Give each team a task to complete and remind each member of the roles they are expected to serve to assist the others in completing the task; 4) Intervene to ensure full participation; 5) Provide an opportunity for the groups to report back to the class at the end of the assigned time.
197. Decide which of the following statements correctly works as the thesis statement of an introductory paragraph. a. Data suggests that the industrialized nations are in fact making considerable progress in reducing their levels of pollution and other environmental damage; however, developing countries such as China and India are industrializing rapidly, and it is widely believed that this industrialization is likely to have significantly negative environmental effects. b. The need to limit human impact on the environment and to avert catastrophic environmental change is unquestionably the most significant long-term challenge facing policymakers today. c. This essay will argue that it is possible for today's developing countries to industrialize without serious environmental compromises, but only if developed countries are willing to provide financial and political assistance.
198. Create coherence by arranging the random statements of a paragraph: 1. In these rural areas 70 percent of the available agricultural land suffers from dryness and lack of irrigation. 2. As a result, the crops are usually marginal with hardly enough production to feed the farmer's family. 3. The first fact to consider is that over 60 percent of the population live in rural communities. 4. In my country, one of the most urgent problems facing the government today relates to agriculture. 5. And without irrigation, agricultural production is wholly dependent on the uncertain rainfall. 6. To understand the nature of this problem one has to look at some of the facts. a. 4 - 5 - 1 - 3 - 6 - 2 b. 2 -6 - 5 - 1 - 3 – 4 c. 4 -6 - 3 - 1 - 5 – 2.
199. Select the option that encloses the main variables involved in language learning which must be primarily taken into account in the process of EFL curriculum development and syllabus design: a. the environment analysis, the learners’ needs, and the four language skills. b. the teacher, the learners, and the role of instructional materials. c. the learning outcomes, the social and contextual constrains. .
200. __________limits the topic of the paragraph to one definite idea or one aspect of the topic that represents a particular idea, feeling, or opinion. It must not be too broad; it must be specific enough for the subject to be discussed within one paragraph. a. The concluding sentence b. The controlling idea c. The topic sentence.
201. In assessing grammar and vocabulary, the deletions in exercises are made to test one or more areas of grammatical knowledge, such as vocabulary andgrammatical ability. The test-taker must choose the appropriate response for the deletion based on the context in which the language is presented. This definition best describes: a. Information gap tasks b. Gap-filling tasks c. Dialogue-completion tasks.
202. Analyze the following case: Mr. Castle’s kindergarten students know exactly what to expect when he starts singing, “Time to clean up.” They immediately begin to put their materials away. They seem to shift into high gear when they see their teacher pick up a book and go to sit in the rocking chair. They all know its story time. They quickly clean up and go sit on their carpet. They love to hear Mr. Castle read stories. What strategy is Mr. Castle using in his class? a. Manipulatives strategies. b. Predictable routines and signals. c. Visual scaffolding.
203. Complete: “The importance of knowing how to _____ lies on the fact that this is the main step to start a successful process of investigation” a. convey reflections b. define a research problem c. validate information.
204. Choose the option that best completes the following statement: A good________facilitates the reading, understanding, and composing of a research study. It can be determined by examining the headings and by looking for the process steps of research. a. purpose statement b. Structure of the study c. Research.
205. Decide which of the following statements correctly works as the general statement of an introductory paragraph. a. This essay will argue that it is possible for today's developing countries to industrialize without serious environmental compromises, but only if developed countries are willing to provide financial and political assistance. b. The need to limit human impact on the environment and to avert catastrophic environmental change is unquestionably the most significant long-term challenge facing policymakers today. c. Data suggests that the industrialized nations are in fact making considerable progress in reducing their levels of pollution and other environmental damage; however, developing countries such as China and India are industrializing rapidly, and it is widely believed that this industrialization is likely to have significantly negative environmental effects.
206. If test measures what it proposes to measure and offers meaningful information about a student´s ability, so this test is: a. Valid b. Accountable c. Practical.
207. Choose the answer that best completes the following sentence: A reliable test ________________________. a. demonstrates the degree of correspondence of the characteristics of a given language test task to the features of a target language task. b. is consistent and dependable. If the same test is given to the same student or matched students in two different occasions the test should yield similar results. c. measures what it proposes to measure. It relies as much as possible on performance. .
208. A syllabus is ____________________ a. a process by which a curriculum designer identifies the students´ needs. b. a detailed description of teachers and learners´ experience during a course. c. a list of content areas which are to be taught in a particular course.
209. When planning a listening and speaking programme for beginners the director should be aware that …. a. there should be regular opportunities for increasing amounts of meaning-focused listening input early in the course. b. Meaning-focus output is the desire of the students and should be tackled head on at the earliest opportunity. c. fluency will only come from speaking with native speakers – a preference should be given to contracting them to work in the programme.
210. Which strategy uses this example: To show a cup while saying: this is a cup, we use it for keeping liquid. a. Multiple intelligences strategies b. Manipulative strategies c. Modeled Talk.
211. Read the following statements and choose the option that is a potential research problem. a. Students feel frustrated when teachers use traditional methods. b. Students feel motivated when using innovative applications in their devices. c. Students find critical thinking activities more appealing .
212. Select the correct statement to identify how to use "what is it?" activity. a. The teacher prepares general knowledge questions, questions, incomplete statements, or true/false statements that the learners will hear and try to complete. b. It involves using listening and often questioning to fill a matrix with information. c. The teacher describes something or someone, and the learners have to decide what is being described. .
213. Read the given paragraph, and chose from ‘a’ to ‘c’ the statement that makes the best topic sentence for the paragraph. [Topic sentence.] On Sunday, unlike the other days of the week when he works, my dad spends the whole day with me watching football on TV. We even eat lunch together while watching. The highlight of the day is watching the Dolphins game. Dad and I get so excited, we yell and cheer together. [Concluding sentence] a. Sunday is my favorite day because I spend the day watching football with my dad. b. Sunday is my favorite day because my father and I do chores around the house. c. Sunday is my favorite day because I my father stays late at work.
214. The results of a needs analysis will generally consist of information taken from several different sources and summarized in the form of ranked list of different kinds like: a. Lists of the most frequent words and expressions learners need to know. b. Lists of principles for organizing and sequencing language content c. Lists of perceived difficulties with different aspects of linguistic use.
215. Which of the following activities can be used to make students focus on extensive reading? a. The teacher asks students to a book of their preference, then he/she asks them to provide the meaning of words in L1 or L2 in the margin next to the line containing an unknown word. b. The teacher gives the students several sentences that contain the main ideas if a story. Each sentence can become the main sentence in a paragraph. Then the students add description and more details. c. The teacher gives the learners the name of something, and they must describe it using the plan. He gives a few new words if they are needed in the description.
216. Select the questions that you will use if you are focusing on needs about genres and discourse types as well as sociolinguistic skills. a. Are the learners happy with the course?/ What content matter will the learner be working with? b. What will the language be used to do? / What language uses is the learner already familiar with? c. What activities will I get the learners to do today?/ Should learners be doing homework?.
217. Its purpose is more notetaking than record-keeping. According to the statement we can point out to: a. An interview b. An observation c. A questionnaire.
218. Analyze the following situation. Which strategy is the teacher applying? As Miss Alvarado prepares her lessons for the next week, she also gathers concrete objects to support the students´ understanding. Besides; her school is near to a little park where she will be able to take the students on a walk to connect language acquisition with the surrounding environment. a. Realia Strategies b. Total Physical Response (TPR) c. Integrated Curriculum Projects.
219. Decide which of the following statements correctly works as the hook of an introductory paragraph. a. Data suggests that the industrialized nations are in fact making considerable progress in reducing their levels of pollution and other environmental damage; however, developing countries such as China and India are industrializing rapidly, and it is widely believed that this industrialization is likely to have significantly negative environmental effects. b. This essay will argue that it is possible for today's developing countries to industrialize without serious environmental compromises, but only if developed countries are willing to provide financial and political assistance. c. The need to limit human impact on the environment and to avert catastrophic environmental change is unquestionably the most significant long-term challenge facing policymakers today.
220. Giving clear instructions is vital for students. Which of the following is the best way to make sure teacher’s instructions were. a. Waiting for students’ questions after giving the instruction. b. Asking students to provide with some examples. c. Receiving tasks the way they were understood.
221. Choose the option that best fits the definition below. ____________ is designed to develop basic communication skills. The development stages are: (1) Comprehension (preproduction), (2) Early Production, and (3) Speech Emergence. a. Language Experience Approach b. Total Physical Response c. Natural approach .
222. Decide which of the following statements correctly works as the thesis statement of an introductory paragraph. a. The need to limit human impact on the environment and to avert catastrophic environmental change is unquestionably the most significant long-term challenge facing policymakers today. b. Data suggests that the industrialized nations are in fact making considerable progress in reducing their levels of pollution and other environmental damage; however, developing countries such as China and India are industrializing rapidly, and it is widely believed that this industrialization is likely to have significantly negative environmental effects. c. This essay will argue that it is possible for today's developing countries to industrialize without serious environmental compromises, but only if developed countries are willing to provide financial and political assistance.
223. Which of the following statements is a correct definition of strategy? a. A strategy is a careful plan or method for achieving a general goal usually over a short period of time. b. A strategy is a plan of action or policy designed to achieve a very specific aim. c. A strategy generally involves setting goals, determining actions to achieve the goals, and mobilizing resources to accomplish the actions.
224. Read the following statement and choose the information that correctly completes it: There are some measures of central tendency; one of them is the mean, which is ___. a. the score that appears most frequently in a list of scores b. the middle score among all scores c. the total addition of the scores divided by the number of scores.
225. Within the Extensive Reading tasks, a teacher may ask the students to __________. a. find and circle/underline all irregular verbs b. find all phrasal verbs used in a text c. write a short summary of each paragraph of reading.
226. LANGUAGE TESTING Informal assessment ______________________. a. is nonjudgmental and ultimate decisions about students’ performance are not made. Here performance is elicited without recording results. b. measures what a student has learned by looking back and taking stock of how well that student has accomplished objectives. c. constitutes a set of procedures constructed to give the teacher and student an appraisal of student achievement.
227. LANGUAGE TESTING If a test is easy to construct, administer, score, and interpret, we speak about its _____. a. reliability b.practicallity c. validity.
228. Read the following research topic and identify which research design it is. The reasons for students not attending to tutorials. a. Quantitative research design b. Mixed research design c. Qualitative research design.
229. Decide whether these statements are examples of introductory paragraphs, body paragraphs, or concluding paragraphs. 1. The main reason for people being unhappy with censorship in any form comes from their mistaken belief that an individual's private actions have no effect on others 2. A related reason for dissatisfaction with censorship appears to be a false notion of the idea of freedom of expression of ideas. a. Introductory paragraphs b. Body paragraphs c. Concluding paragraphs .
230. Which of the following are types of reading assessment: a. Form-completition, Cloze selection, and Grammatical transformation. b. Perceptive, Interactive, and Extensive. c. Imitative and Intensive.
231. Read the following statement and choose the information that There are some measures of central tendency; one of them is the mode, which is ______________. a. the score that appears most frequently in a list of scores b. the total of the scores divided by the number of scores c. the middle score among all scores.
232. Select the item that states the aspects that English teachers must consider when selecting authentic reading material for a course. a. The needs of the community, the educational policies and the teaching resources. b. Approach, the financial cots, the location of the educational institutions. c. Suitability of content, exploitability, readability and presentation.
233. _________play a significant role in EFL teaching and learning since they provide useful ready-made material to both teachers and students. a. Curriculum and materials b. Course books c. Standards and principles.
234. Read the following information and then choose the correct response. Informal assessment can take a number of forms. These forms can be; for example, ___ a. A proficiency test that tests global competence in a language and that is not limited to any one course, curriculum, or single skill in the language; rather, it tests overall ability is a form of informal assessment. b. Diagnose aspects of a language that a student needs to develop or that a course should include. c. Say to your students expressions like “Good work”; “Did you say can or can’t?”; “I think you meant to say you broke the glass, not break the glass”. Another example is when teachers design tasks to elicit performance without recording results. .
235. Choose the option that best completes the following statement: A good _________ facilitates the reading, understanding, and composing of a research study. It can be determined by examining the headings and by looking for the process steps of research. a. Research b. Structure of the study c. purpose statement.
236. Which of the following techniques can be used when students have problems at the moment of understanding high frequency vocabulary in a text? a. Ignore the word. b. Replace the word in the text with a low frequency word. c. Make a glossary before the learners read the text. .
237. EDUCATIONAL RESEARCH Read the following research question and choose the research design it belongs to: How well do homeschoolers who attend college do compared to the ones who attended traditional schools? a. Mixed research design b. Quantitative research design c. Qualitative research design .
238. Curriculum implies all the issues related _____________, which are conceived as a coherent whole with a specified purpose. a. to educational policies and laws for English teaching in schools and high schools b. to planning, implementation and evaluation of a series language learning events c. to teaching, monitoring and assessing English teaching.
239. Select the activity that helps students work on intensive reading. Read the student’s introduction to an essay. Then answer the following questions: a. - Does the writer spark the reader’s interest at the very beginning? - Does the writer go from broad general statements to specific statements? b. Using the information that you have read about cause-and-effect essays, write a well-written essay focused on the positive or negative effects of social media. c. Work in groups of three students and share information about one of the books you read the last week. Then prepare an oral book report for your classmates.
240. Needs analysis can play a major role in determining the ___________ a. number of students for the course. b. steps of the environment analysis. c. content of courses, particularly for language items.
241. When the teacher aims at assessing acceptable pronunciation only, _________should be applied. a. paraphrasing b. dictation or cloze testing format c. imitative speaking techniques.
242. Select the option that encloses the main variables involved in language learning which must be primarily taken into account in the process of EFL curriculum development and syllabus design: a. the teacher, the learners, and the role of instructional materials b. the learning outcomes, the social and contextual constrains c. the environment analysis, the learners’ needs, and the four language skills.
243. INTRODUCTION TO APPLIED LINGUISTICS The field o Psycholinguistics deals with: a. Language acquisition, comprehension, and production b. Analysis of language variations and language changes c. Classification of world languages and their use by speakers .
244. INTRODUCTION TO APPLIED LINGUISTICS Technical vocabulary, words or expressions such as LO L, IMHO, TOEFL, AGONAL, FLIPPED LEARNING, GIVE ME THE VITALS, etc. are used by a profession or close groups that are difficult for others to understand outside the context. These words represent examples of language variation known as: a. Jargon b. Non-standard Language c. Slang.
245. INTRODUCTION TO APPLIED LINGUISTICS The “universal grammar” theory implies that there is a connection between speech and human relations and human understandings of the world. True False.
246. INTRODUCTION TO APPLIED LINGUISTICS The notion of this discipline is necessary for areas of cross-cultural communication, for correct use of language in general, because it is considered the goal of language education, and it is necessary element in efficient classroom practice. It refers to communicative competence. True False.
247. INTRODUCTION TO APPLIED LINGUISTICS Applied Linguistics is usually defined by linguists as the academic discipline that is concerned with the relation of knowledge about language in general to decision making in the real world, and it is because it explands the knowledge about the world languages. True False.
248. INTRODUCTION TO APPLIED LINGUISTICS Choose the most suitable application of Sociolinguistics for education: a. It explains the process of language acquisition and issues related with bi - and multilingualism. b. It focuses on the role of language behavior for education and language acquisition c. It provides description of language variations and explains the changes of language through time.
249. INTRODUCTION TO APPLIED LINGUISTICS The words “rise” and “ascend” are synonyms. However, in everyday speech you would probably choose the word “to rise” rather than “to ascend”. According to Communicative competence theory, your choice depends on the frequency of the occurrence of each word. This is known as: a. attestedness b. feasibility c. appropriateness.
250. INTRODUCTION TO APPLIED LINGUISTICS “Adjectives in English are never pluralized", “Splitting infinitives is wrong”, “Modal verbs do not change”. These statements represent: a. restricted code b. descriptive approach to language learning c. prescriptive approach to language learning.
251. Read the analogy and answer the question that follows. “Blogging is another example that is best compared to writing a personal diary entry and posting it for the world to read.” What point is the author trying to make? a. a personal diary entry and the world b. The world is much bigger than you realize. c. Personal information is revealed through blogging.
252. Identify the relationship between the words in the analog, and then choose the option that follows the pattern. For example: STARVING: HUNGRY a. towering: cringing b. neat: thoughtful c. depressed: sad.
253. Identify the relationship between the words in the analog, and then choose the option that follows the pattern. For example: tree: leaf a. flower: petal b. roots: soil c. garden: grass.
254. Read the article. Choose the best answer for each question. 1 Bali, one of Indonesia's 17,000 islands, conjures up images of perfect, golden beaches and azure waters. This description would be accurate, except for the plastic debris which litters the shores. In 2018, the issue of ocean pollution here received worldwide attention when a scuba diver called Rich Horner posted a video of himself swimming in a sea of plastic. The clip was horrifying, and it quickly went viral. 2 Two Dutch-Indonesian sisters, Melati and Isabel Wijsen, were already aware of the scale of the problem. Having grown up in Bali, they knew that plastic contamination was a major concern. With time they became convinced that they could be instrumental in bringing about change. To solve the problem, they believed it was important to harness the power and enthusiasm of young people. They decided to launch a movement, which they called “Bye Bye Plastic Bag”. 3 The sisters began collecting signatures to support their cause, and eventually got over 100,000 people to register their concern. In 2017, they took their mission to the international stage by giving a moving speech at the United Nations in New York City. In the talk, they showed heartbreaking photos of animals entrapped in plastic and of stunning natural landscapes which had been completely soiled. The point they were making was simple: single- use plastics such as shopping bags, straws and Styrofoam containers should be banned. 4 Gradually, Melati and Isabel's efforts gained momentum. "We didn't want to wait until we were older to stand up for what we believe in," they said, stressing that kids have the ability to effect change. In late 2018, they achieved a major success when the governor of Bali announced that single- use plastics would be forbidden on the island. Not only that, but the local government committed to a major marine cleanup effort. 5 Young people can lead the way, the sisters believe. They say they won't rest until visitors to the island paradise hear this question at the airport: "Welcome to Bali! Do you have any plastic bags to declare?" Rich Horner’s video is proof that there is still more work to be done. However, the sisters’ work suggests that ambitious conservation efforts are by no means impossible. The article focuses mostly on: a. two sisters' success in banning plastics b. a horrifying video of polluted waters c. plastic waste in Indonesia's capital.
255. Read the following passage, then choose the option that includes true statements: Opera refers to a dramatic art form, originating in Europe, in which the emotional content is conveyed to the audience as much through music, both vocal and instrumental, as it is through the lyrics. By contrast, in musical theater an actor's dramatic performance is primary, and the music plays a lesser role. The drama in opera is presented using the primary elements of theater such as scenery, costumes, and acting. However, the words of the opera, or libretto, are sung rather than spoken. The singers are accompanied by a musical ensemble ranging from a small instrumental ensemble to a full symphonic orchestra. a. It is pointed out in the reading that opera is a drama sung with the accompaniment of an orchestra. We can understand from the reading that orchestras in operas can vary considerably in size. It is stated in the reading that music in musical theater is not as important as it is in opera. b. It is pointed out in the reading that opera has developed under the influence of musical theater We can understand from the reading that people are captivated more by opera than musical theater It is stated in the reading that acting and costumes are secondary to music in musical theater c. It is pointed out in the reading that opera is the most complex of all the performing arts We can understand from the reading that there is argument over whether the music is important or the words in opera It is stated in the reading that opera doesn't have any properties in common with musical theater.
256. Read the article. Choose the best answer for each question. 1 Bali, one of Indonesia's 17,000 islands, conjures up images of perfect, golden beaches and azure waters. This description would be accurate, except for the plastic debris which litters the shores. In 2018, the issue of ocean pollution here received worldwide attention when a scuba diver called Rich Horner posted a video of himself swimming in a sea of plastic. The clip was horrifying, and it quickly went viral. 2 Two Dutch-Indonesian sisters, Melati and Isabel Wijsen, were already aware of the scale of the problem. Having grown up in Bali, they knew that plastic contamination was a major concern. With time they became convinced that they could be instrumental in bringing about change. To solve the problem, they believed it was important to harness the power and enthusiasm of young people. They decided to launch a movement, which they called “Bye Bye Plastic Bag”. 3 The sisters began collecting signatures to support their cause, and eventually got over 100,000 people to register their concern. In 2017, they took their mission to the international stage by giving a moving speech at the United Nations in New York City. In the talk, they showed heartbreaking photos of animals entrapped in plastic and of stunning natural landscapes which had been completely soiled. The point they were making was simple: single- use plastics such as shopping bags, straws and Styrofoam containers should be banned. 4 Gradually, Melati and Isabel's efforts gained momentum. "We didn't want to wait until we were older to stand up for what we believe in," they said, stressing that kids have the ability to effect change. In late 2018, they achieved a major success when the governor of Bali announced that single-use plastics would be forbidden on the island. Not only that, but the local government committed to a major marine cleanup effort. 5 Young people can lead the way, the sisters believe. They say they won't rest until visitors to the island paradise hear this question at the airport: "Welcome to Bali! Do you have any plastic bags to declare?" Rich Horner’s video is proof that there is still more work to be done. However, the sisters’ work suggests that ambitious conservation efforts are by no means impossible. What conclusion does the article support? a. Bali should fine tourists for using plastic bags. b. The country is no longer a good tourist destination. c. Young people can make a difference.
257. Read the following passage, then choose the option that includes true statements. Dolphins are regarded as the friendliest creatures in the sea and stories of them helping drowning sailors have been common since Roman times. The more we learn about dolphins, the more we realize that their society is more complex than people previously imagined. They look after other dolphins when they are ill, care for pregnant mothers and protect the weakest in the community, as we do. Some scientists have suggested that dolphins have a language but it is much more probable that they communicate with each other without needing words. Could any of these mammals be more intelligent than man? Certainly the most common argument in favor of man's superiority over them that we can kill them more easily than they can kill us is the least satisfactory. On the contrary, the more we discover about these remarkable creatures, the less we appear superior when we destroy them. a. It is clear from the passage that dolphins don't want to be with us as much as we want to be with them One can infer from the reading that communication is the most fascinating aspect of the dolphins b. It is clear from the passage that dolphins are proven to be less intelligent than once thought. One can infer from the reading that dolphins have skills that no other living creatures have such as the ability to think c. It is clear from the passage that dolphins have a reputation for being friendly to humans One can infer from the reading that dolphins have some social traits that are similar to those of humans.
258. Read the paragraph below and select the sentence that is not about the main idea? (1) Antiques Roadshow is a popular television show. (2) The show travels to different cities. (3) The show is not very funny. (4) The guests are regular people. (5) They bring their special possessions to the show. (6) They tell stories and ask questions. (7) You can see similar shows in other countries around the world. a. 3 b. 4 c. 6.
259. Read the text. Choose the correct group of words to complete fill the gaps. The report also found that people in Iraq are the kindest to strangers. Eight in 10 Iraqis are reported to have helped someone they don't know in the past year. The BBC commented ………… how significant this statistic was, the turmoil Iraq has been in for years because of war. It stated that: "It is the kindness of Iraqis…to complete strangers in the of years of conflict and terrible violence which stands out in the list." John Low, CEO of the Charities Aid Foundation, said: "The generosity of people, even in countries suffering from disaster and turmoil, is ………… humbling." He added: "Unconditional gifts of time and money are a life-changing for good in the world." a. on, force, face, truly, given b. truly, given, on, face, force c. on, given, face, truly, force.
260. Read the following sentences; are these examples of effective topic sentence? Japanese culture is fascinating to study because its family traditions are so different from American traditions. I will talk about the reasons for the popularity of reality television shows. True False.
261. Read the introductory paragraph and choose the option that presents the correct information. (A) My brother is deaf, yet he is a very happy member of society. (B) While many hearing people at first feel sorry for him when they meet him, they only feel sorry for him because they don’t know him. (C) If they knew him like I do, they would know that he accepts his deafness, and he is proud to be a member of the deaf community. (D) Members of the hearing community would be wise to follow the lead of deaf people in defining their own groups, their own limitations, and their own acceptance of themselves. a. Sentence B states why the writer’s opinion matters. Sentence C hooks the reader with something surprising. Sentence A is the thesis statement. b. Sentence A states why the writer’s opinion matters. Sentence B hooks the reader with something surprising. Sentence D is the thesis statement. c. Sentence C states why the writer’s opinion matters. Sentence A hooks the reader with something surprising. Sentence D is the thesis statement.
262. ENGLISH PHONOLOGY Choose the couple words that represent minimal pairs: a. haze - blaze rows- rose hear – hair fair- file b. nose- noise could–good would–good coat-goat c. would – wood knot- not sent – scent lake- pray.
263. COMMUNICATIVE GRAMMAR Read the conversations. Choose the correct subordinator or transition word for each blank space. Hannah: I always take my own reusable bag to the store. Jessica: I try to do that too, _______ my sister never does. a. whereas b. however c. just as.
264. COMMUNICATIVE GRAMMAR Choose the correct option to complete the sentence: Many cultures share similar new year traditions; for example, wearing yellow clothes, eating grapes, ____________debts, lighting candles, etc. a. getting together b. giving out c. paying back .
265. COMMUNICATIVE GRAMMAR Select the correct verb forms to complete the statement below: We met our friend Carol, while she ______________ Architecture at Harvard University. By the time we got married, Carol ___________her studies. a. studies / is completing b. is studying /completes c. was studying /had completed.
266. SYNTAX AND TEACHING GRAMMAR It refers to the hierarchical arrangement of the constituent words and phrases of a sentence to form well-formed and meaningful structures. It is a formal representation of the grammatical structure of a sentence a. Constituency b. Active clause c. Construction.
267. PSYCHOLOGY AND LANGUAGE LEARNING The Behaviorist perspective mentions that Chomsky´s concluded that children´s minds are not blank slates to be filled by imitating languages they hear in the environment. True False.
268. PSYCHOLOGY AND LANGUAGE LEARNING Clarification requests, Elicitation and Repetition are some techniques that teachers use as Instructional approach. True False.
269. PSYCHOLOGY AND LANGUAGE LEARNING Fossilization is the term used to describe when some features of learner language stop changing over time? True False.
270. TEACHING TECHNIQUES Analyze the following case. Then, answer the question: In order to support the comprehension of a tough text, the teacher reads it to the whole class adding visual support, periodic paraphrasing; and/or rewriting. Through this, the students get actively involved in the activity because they are part of it; so they are more motivated to listen carefully as the teacher reads. What strategy for literacy development is the teacher applying? a. Moving into Reading b. Guided Reading c. Read-aloud plus.
271. TEACHING TECHNIQUES Analyze the following application example and complete the statement below: Having students talk about their classroom work gives them practice in using academic language, demonstrating their growing knowledge with teacher support and scaffolding. It is an example of the language development strategy……... a. Collecting and processing words b. Reporting back c. Scaffolding English writing.
272. TEACHING TECHNIQUES Which of the following statements DOES NOT correspond to advantages of using realia in the classroom? a. Real objects can be seen, felt and manipulated, which is a powerful way to connect vocabulary to real life. b. Real objects are highly effective in supporting comprehension and vocabulary development in older students, especially those who are English learners. c. The use of real objects conveys meaning in a way that no photograph or illustration .
273. TEACHING TECHNIQUES Choose the option that best fits the strategy below. The strategy, applied during a course, in which the teacher observes and documents the students’ performance on academic tasks rather than on their fluency in English to identify weak students by using Rubrics and checklists is called____________. a. Portfolio assessment b. Performance sampling c. Anecdotal records.
274. LANGUAGE TESTING Informal assessment _______________. a. is nonjudgmental and ultimate decisions about students’ performance are not made. Here performance is elicited without recording results b. constitutes a set of procedures constructed to give the teacher and student an appraisal of student achievement. c. measures what a student has learned by looking back and taking stock of how well that student has accomplished objectives.
275. LANGUAGE TESTING If a test is easy to construct, administer, score, and interpret, we speak about its______________. a. reliability b. practicality c. validity.
276. LANGUAGE TESTING If a test contains natural language, meaningful and relevant topics and contextualized items that replicate real- world tasks, we say that this test is___________. a. authentic b. reliable c. valid.
277. LANGUAGE TESTING Analyze the following example and decide to which category of speaking performance it belongs. The task includes interaction and test comprehension but at the limited level of short conversations, standard greetings, small talks, requests, and comments. a. Intensive b. Responsive c. Interactive.
278. METHODOLOGY AND DIDACTICS From the following which one is an example of intensive listening? a. Listening to news stories and biography information about famous international sports stars. b. Listening to a radio announcement about an event and filling out a table or a series of blanks with words from the listening. c. English reviews of movies or TV shows that you've seen in your language.
279. METHODOLOGY AND DIDACTICS Social relationships and collaborative opportunities play an important role in supporting teachers in managing disruptive behavior in their classroom. Which is one strategy that teachers can use to decrease misbehavior in class? a. Counting with therapist’s help and principal intervention to correct students’ behavior and prevent future discipline problems. b. Setting up good relationships with colleagues, previous students’ teachers and parents for continuous guidance and support. c. Keeping a record of students’ discipline to find out the specific punishment students who misbehave should be given.
280. METHODOLOGY AND DIDACTICS Which of the following question types cannot be used when making students focus on comprehension of the text? a. Sentence completion b. Glossing c. Multiple-choice sentences.
281. METHODOLOGY AND DIDACTICS What is the focus of the following activity? The teacher asks students to recall a list of items, actions, or events explicitly stated in the text or the sequence of several actions or events explicitly stated in the text they have already read. a. Literal comprehension of the text b. Drawing inferences from the text c. Responding critically to the text.
282. EDUCATIONAL RESEARCH Creswell identifies and proposes some definitions of the parts of a research. Choose the option that does not correspond to his statements. a. A research topic is the broad subject matter addressed by the study. b. A research problem is a general educational issue, concern, or controversy addressed in research that narrows the topic. c. The literature review is the major intent or objective of the study used to address the problem .
283. EDUCATIONAL RESEARCH Which of these statements is correct? a. The skills needed for quantitative researchers include the ability to write detailed passages and analyzing words from participants in the study. b. Regardless of the approach, the amount of time needed for data collection can only be estimated by examining similar studies. c. Researchers need to create a budget and obtain advice from other, experienced researchers about whether the anticipated expenses are realistic.
284. EDUCATIONAL RESEARCH Creswell (2012) asserts “The researcher must have access to people and sites and possess the time, resources, and skills to study the problem. The study needs to contribute to knowledge and practice. There also needs to be a match between the research problem and the qualitative approach chosen for the study." True False.
285. EDUCATIONAL RESEARCH The importance of knowing how to define a Research Problem lies on the fact that this is the main step to start a successful process of investigation. Once identified the problem the researcher can state through clear ideas at the "statement of the problem". True False.
286. EDUCATIONAL RESEARCH Creswell (2012) states that a research proposal is a completed study that reports an investigation or exploration of a problem, identifies questions to be addressed, includes the collection and analysis of data, and advances an interpretation of the data. True False.
287. CURRICULUM AND MATERIALS Choose the option that contains the correct information. a. Materials development means creating, organizing, or adopting/adapting materials and activities so that students can achieve the objectives of a course. b. The use of basic or supplementary material as well as technological resources impacts negatively the effectiveness of the teaching learning process. c. Materials need to have an underlying instructional philosophy, approach, method and technique which suit the students and their needs since some of them are appropriate for all the classes and students' background.
288. CURRICULUM AND MATERIALS ____________ are variables that will significantly impact on decisions about teaching materials. Many teachers are bound by a mandated curriculum defining the content, skills and values to be taught. a. The resources and facilities b. Personal confidence and competence c. The curriculum and the context.
289. CURRICULUM AND MATERIALS Which of the following statements is true regarding curriculum? a. Curriculum is a formal document which outlines the aims, the selection and grading of contents, teaching materials, tasks and activities, etc. to be taught in a specific course, unit of study or teaching subject. b. Curriculum comprises teachers and students’ believes in terms of their experience and personal view of language, teaching, learning, selectionn of content as well as the influence of social and political contexto on learning outcomes. c. Curriculum is a general plan which comprises educational foundations and guidelines of what, why and how students should learn. In other words it contains, the knowledge, skills, values and attitudes expected to be learned by students.
290. ________________ measure learners’ ability within a classroom lesson, unit, or even total curriculum. a. Proficiency tests b. Achievement tests c. Diagnostic tests.
291. Teaching pronunciation depends directly on the student´s proficiency level; therefore, the teacher has to plan to teach pronunciation in a progressive way. Analyze the options and select the one that reflects the correct order in which pronunciation must be taught to EFL students. a. At the beginning level, the teacher focuses student´s attention on prosodic elements such as intonation in different sentence types, voice quality, rhythm and so on.; at the intermediate level, the attention is centered on teaching the stress of words and sentences; and, at advanced level, pronunciation goals focus on articulation of vowels and consonants. b. At the beginning level, the teacher focuses student´s attention on the correct articulation of vowels and consonants; at the intermediate level, the attention is centered on prosodic elements such as intonation in different sentence types, voice quality, rhythm and so on; at advanced level, pronunciation goals focus on stress of words and sentences. c. At the beginning level, the teacher focuses student´s attention on the correct articulation of vowels and consonants; at the intermediate level, the attention is centered on teaching the stress of words and sentences; and, at advanced level, pronunciation goals focus on prosodic elements such as intonation in different sentence types, voice quality, rhythm and so on.
292. What is the meaning of the following sentence? My teacher made me rewrite the essay. a. I didn’t write the essay again. b. I finished the essay. c. I wrote the essay again.
293. What strategy should an EFL teacher apply in the following situation? If a group of students presents serious word stress, intonation, and rhythm pronunciation problems in class. a. Identify specific pronunciation features that cause problems for learners. b. Focus on developing learners’ communicative competence in English. c. Make learners aware of the prosodic features of the target language. .
294. The phenomenon of Language Transfer refers to errors produced by second/foreign language learner as a result of: a. lack of exposure to target language or appropriate language mode b. borrowing patterns from student´s mother language and using words and already known structures c. lack of reinforcement of correct language production and constructive feedback.
295. Sociolinguistics is a discipline that focuses on the study of how language and social factors are related. It basically explains: a. the cognitive processes that enable human beings to acquire languages (native, second, foreign), construct a grammatically correct and meaningful sentences, and understand utterances, words, text, etc. b. how learning environment influences the process of language acquisition in general c. how people of different social status, gender, age, race, ethnicity, speak and how their speech changes in different situations.
296. Which of the following is not a characteristic of the innateness perspective? a. The human brain is ready naturally for language in the sense when children are exposed to speech; certain general principles for discovering or structuring language automatically begin to operate b. Before children can use linguistic structures, they need first to have developed the conceptual ability to make relative judgments c. The presence of Universal Grammar in the brains of children allows them to deduce the structure of their native languages from “mere exposure”.
297. Semantic units of language in use include Text and Discourse, both are fundamental in coherent communication, which means that they… a. are used to form correct grammatical structures. b. are semantically and pragmatically coherent in real communication. c. are specific structures used to express a single message.
298. Please choose the correct answer from the choices presented. Universal or global culture is used to refer to_______________. a. human civilization separated from nature and nurture. b. the behaviors and beliefs characteristic of particular social, ethnic, religious or political communities. c. anything from fine arts to soap operas, soda commercials and commodities.
299. INTRODUCTION TO APPLIED LINGUISTICS Examples of incorrect use of language such as “beautifuls flowers,” How many years old do you have?”, "I see many fishes” represent: a. Non -standard language b. Linguistic competence c. Language transfer.
300. INTRODUCTION TO APPLIED LINGUISTICS Choose the option that belongs to the category of language- related problems: a. Description of world languages b. Comparison and classification of world languages c. Machine translation and analysis of language codes.
301. COMMUNICATIVE GRAMMAR 1. Choose the correction option to complete the sentence Lily: “It seemed to be a nice morning. I was getting ready for my departure when I___________I had lost my passport. I was shocked!” a. realized b. had realized c. have realized .
302. COMMUNICATIVE GRAMMAR Analyze the example and choose the correct option: I don’t have to worry about my meeting as ______________________ a. I’m already finished that report b. I already have finished that report c. I’ve already finished that report.
303. COMMUNICATIVE GRAMMAR Read the following statement and choose the correct option. I will get that book for you! a. Prediction b. Request c. Promise.
304. COMMUNICATIVE GRAMMAR Read the sentence and choose the correct description: You had better get going and talk to your supervisor before he leaves. In this case, “had better” expresses: a. a good idea b. soften advice c. strong advice.
305. COMMUNICATIVE GRAMMAR Select the correct verb form to complete the following statement: I left the band when I found out that our manager (take)_________our money for a long time. a. had been taking b. is taking c. takes .
306. COMMUNICATIVE GRAMMAR Complete the question below by choosing the correct tag. They are not in the concert anymore. ____________________? a. are they b. aren’t they c. are they not.
307. COMMUNICATIVE GRAMMAR Choose the adjective clause that best fits the following sentence: In 1954, MacLennan published a collection of essays. Dorothy Duncan edited this collection for him. a. In 1954, MacLennan published a collection of essays whose Dorothy Duncan edited for him. b. In 1954, MacLennan published a collection of essays that Dorothy Duncan edited for him. c. In 1954, MacLennan published a collection of essays when Dorothy Duncan edited for him.
308. COMMUNICATIVE GRAMMAR Read the following sentences and choose the correct relative clause. The new teacher is a very interesting person. / She comes from England. a. The new teacher who momes from England that is a very interesting person. b. The new teacher who comes from England, is a very interesting person. c. Who comes from England is a new teacher very interesting person. .
309. COMMUNICATIVE GRAMMAR Choose the phrasal verb that has the same meaning as the boldfaced word in the passage below. Are you gaining weight? If you’re like most people, you need to reduce your food portions and get off the couch. Make a plan to eat right and get regular exercise, and stick to it. a. cut down on b. come up with c. end up with.
310. COMMUNICATIVE GRAMMAR Choose the phrasal verb that has the same meaning as the boldfaced word in the sentence below. Don’t distract me. I’m trying to concentrate. a. look me b. down put me off c. call me off.
311. COMMUNICATIVE GRAMMAR In the following dialogue, there are some modals. Choose the one that is not used in a correct way. Maggie: Hi Tom, have you finished your homework? Tom: Oh hi Maggie. No, I haven’t. Maggie: The deadline is tomorrow you know so you have to submit it tomorrow. Tom: I can’t make it. I haven’t even started it yet. Can we hand it in next week? Maggie: I don’t know. You’ll have to ask Mrs. Robinson about that. I think you ought to finish it by tomorrow. She probably won’t accept projects after tomorrow. a. can b. have to c. ought to.
312. CULTURE AND LITERATURE Which group of people were responsible for bringing Latin into England and therefore into English? a. Mongols b. Christians c. Mormons.
313. CULTURE AND LITERATURE Most classic literature fits into the category of written words. Which of the following fits the same category? a. Songs and Oral stories b. Pictures and Music c. Texts and e-mails.
314. CULTURE AND LITERATURE Literature is important to a cultural group because: a. It is a way to know exactly how everyone felt at that time. b. It always shows the hits and misses of the culture during that period. c. It is a record of the ideas, beliefs and practices of the culture at that time.
315. CURRICULUM AND MATERIALS Curriculum is commonly perceived as the _______________. a. document that contains information about teacher´s training and experience. b. set of processes to follow in order to select instructors for a course. c. combining of subject matter to form a body of content.
316. CURRICULUM AND MATERIALS The following principle “A language course should progressively cover useful language items, skills and strategies” means that _________________. a. teachers should consider individual differences and provide learners with a wide variety of activities to cope all the learners needs. b. the course should have explicit language teaching goals and that there should be some way of ensuring that there is opportunity for the goals to be reached. c. attention must be directed to language features that are more predominant. .
317. EDUCATIONAL RESEARCH A group of researchers at UTPL are carrying out research that will analyze the information that the sample provides in a multiple-choice survey. This survey also contains open-ended questions that allow participants to write their opinion about the research topic. For this research, they will use ________________. a. Experimental Designs b. Ethnographic Designs c. Mixed Methods Designs.
318. EDUCATIONAL RESEARCH The following statement is a good example of a ____________________________. The purpose of this study is to examine the relationship between use of internet communication between teachers and parents in a private school. a. Research topic b. Research problem c. Purpose statement.
319. EDUCATIONAL RESEARCH The following question is an example of a(n) _______ research question. What is creativity for five students at Bayside High School? a. qualitative b. quantitative c. experimental.
320. EDUCATIONAL RESEARCH We can state that a Literature review is _____________ a. exploring a problem and developing an understanding on the issue. b. a revision of the information found in reliable sources. c. evaluating criteria and including your reflections. .
321. ENGLISH PHONOLOGY: PRONUNCIATION Stress placement (the stress shift from one to another syllable) may change the part of speech. It is known as ______function of stress. a. grammatical b. attitudinal c. Pragmatic.
322. ENGLISH PHONOLOGY: PRONUNCIATION The alternation of stressed and unstressed syllables is called: a. pitch b. intonation c. rhythm.
323. ENGLISH PHONOLOGY: PRONUNCIATION Words containing simple vowels are: a. bruise, smile, go, mouse, broke b. assault, peasant, determined, lost, leather c. most, ocean, idea, broke, pool.
324. LANGUAGE TESTING The principle that refers to the logistically, administrative issues involved in making, giving, and scoring an assessment instrument is ____________. a. Test reliability b. Validity c. Practicality .
325. LANGUAGE TESTING ______________refers to the degree to which a test looks right, and appears to measure the knowledge or abilities it claims to measure. a. Washback b. Reliability c. Validity.
326. LANGUAGE TESTING A collection of student´s work that demonstrates his/her effort, progress and achievement in given areas is called: a. Journal b. Portfolio c. Conferences.
327. LANGUAGE TESTING _____________ are used to evaluate students in the process of forming their competencies and skills. a. Formal assessments b. Informal assessments c. Hybrid assessments.
328. LANGUAGE TESTING ______________ involves the production of short stretches of oral language designed to demonstrate competence in a narrow band of grammatical, phrasal lexical, or phonological relationships. a. Intensive speaking b. Imitative speaking c. Interactive speaking.
329. METHODOLOGY AND DIDACTICS This kind of seating arrangement creates military controlled environment, interaction is not present at all, everyone’s attention is directed to the front, it is quite restrictive and students have little opportunity to participate. a. Horseshoe b. Traditional rows c. Separate tables .
330. METHODOLOGY AND DIDACTICS Planning allows teachers to organize all events that are going to happen in the class. Effective lesson sequence facilitates scaffolding that refers to: a. The order in which material is presented, the teacher plans what is intended to happen during each lesson. b. Teachers’ good organization skills, the teacher should know what will be covered in a particular lesson. c. Teachers’ anticipations to any difficulty students might have. Additional support and resources might help student to attain the objective of the lesson.
331. METHODOLOGY AND DIDACTICS Analyze the following situations and determine which one of them show realibility? a. When you deliver a test to the same participant on two occasions, you would be very likely to reach the same conclusions about the participant’s knowledge or skills. b. When you obtain different scores in two instances that you used the test to assess your students. c. When the test should measure what it claims to measure. Items should be closely linked to the test’s intended focus.
332. METHODOLOGY AND DIDACTICS Which of the following is an example of pronominal questions? a. A good reader makes about ten fixations per second. b. A skilled reader makes around fixations per minute. c. What is a saccade?.
333. METHODOLOGY AND DIDACTICS The following activity helps students work on ______________. Read the student’s introduction to an essay. Then answer the following questions: Does the writer spark the reader’s interest at the very beginning? Does the writer go from broad general statements to specific statements? a. intensive reading b. extensive reading c. writing skills.
334. METHODOLOGY AND DIDACTICS Which of the following activities is appropriate for making students practice expeditious reading? a. The teacher gives students an interesting text to read. Then they are required to look for particular name or a particular number included in the text. b. One student is paired with a more proficient reader. Both sit side by side and read the same passage aloud together with the more proficient reader keeping at the same speed as the less proficient reader. The less proficient reader nudges the more proficient reader as a signal that he/she wants to read alone. c. The teacher asks students to silently re-read two texts that have been previously read. While reading students can take notes about the time each reading took so that they have the goal of reading them faster each time. .
335. METHODOLOGY AND DIDACTICS Read the following example about dictation, and then answer the question. Example: Teacher asks students to listen to a short text, the teacher reads twice to the students while they are taking notes. In small groups, they reconstruct a written form of the text from these notes. What kind of dictation activity is related to this example? a. Dictogloss activity b. Monitoring dictation c. Running dictation.
336. METHODOLOGY AND DIDACTICS The type of feedback that makes the speaker take an active role is known as __________________. a. communication b. negotiation c. adoption.
337. METHODOLOGY AND DIDACTICS Complete the following statement: _______is also called interactional speaking because it makes learners produce what they would not normally be able to produce. a. interactive speaking b. informal speaking c. transactional speaking.
338. METHODOLOGY AND DIDACTICS Formal speaking involves taking long turns. This planning can be done in several ways. Choose the correct one: a. The learners should have the opportunity to work through a series of spoken tasks. b. The speaker can look at the ideas that will be presented and find an effective way of organizing them. c. The speaker should pay attention to turning ideas into speech while they speak.
339. METHODOLOGY AND DIDACTICS This effect occurs at the time that the notes are taken. This effect means that the information can be changed from one form to another. It can mean changing from one form of organization of the ideas to another form of organization. a. Encoding effect b. Storage effect c. Interactive effect.
340. PSYCHOLOGY AND LANGUAGE LEARNING Read the definition and choose the correct option: It is based on the desire to know more about the culture and community of the target language group and even a desire to be more like members of that group. a. Integrative motivation b. Instrumental motivation c. Classroom observation scheme.
341. PSYCHOLOGY AND LANGUAGE LEARNING Read the concept below and match with the corresponding name. Choose the correct option It discourage the risk-taking, which is necessary for progress in language learning. This is often considered to be a particular problem for adolescents, who are more self-conscious than younger learners. a. Extroversion b. Anxiety c. Inhibition.
342. PSYCHOLOGY AND LANGUAGE LEARNING Match the following conditions to the correct hypothesis proposed by Krashen Conditions: The acquirer/learner must know the rule The acquirer must be focused on correctness The acquirer/learner must have time to use the monitor. a. The Acquisition-Learning hypothesis b. The Input Hypothesis c. The monitor hypothesis.
343. PSYCHOLOGY AND LANGUAGE LEARNING Read and analyze the following theory, then choose the correct option. “The cognitive development of children is advanced through social interaction with other people, particularly those who are more skilled. In other words, social learning comes before cognitive development, and that children construct knowledge actively.” a. Cognitive Development Theory b. Socio Cultural Theory c. The information Processing Theory.
344. PSYCHOLOGY AND LANGUAGE LEARNING Which of the following characteristics is NOT focused on "Using cooperative rather than competitive goals"? a. Increase the self-confidence of students, including weaker ones. b. Well results working on team-mates, role plays and discourses. c. Grammar exercises on the book, allow students be ready to complex texts.
345. SYNTAX AND TEACHING GRAMMAR In Syntax, hierarchical structures, also called tree diagrams, are used to show the function that each constituent has within a sentence. Hence, constituency is defined as ____________________. a. a node that connects different parts of a grammatical structure. b. the relation among several linguistic units that form a sentence. c. a word or group of words that function as a single unit within a hierarchical structure.
346. SYNTAX AND TEACHING GRAMMAR Analyze the sentence “Bertha told me a lie that really hurt me”. In this syntactic structure: a. The complementizer THAT can easily be omitted without changing the meaning of the sentence. b. The complementizer THAT can be omitted since it introduces additional information to the sentence. c. The complementizer THAT cannot be delated because the meaning of the sentence could change. .
347. SYNTAX AND TEACHING GRAMMAR Types of nouns include countable and collective nouns. The difference between them is that ___________________. a. countable nouns are pluralized by adding “s” or “es” at the end of the word or modifying its spelling like in cases of irregular nouns while collective nouns do not need to use “s” or “es” since the word can be seen as a unit. b. countable nouns are pluralized by adding “s” or “es” at the end of the word or modifying its spelling like in cases of irregular nouns while collective nouns can never be used for plurals. c. countable nouns are pluralized by adding “s” or “es” at the end of the noun or modifying its spelling like in cases of irregular nouns while collective nouns cannot be used for singular.
348. SYNTAX AND TEACHING GRAMMAR Dependent clauses, differing from independent clauses, cannot stand alone since they do not make sense on their own. Hence, they need to be linked to the main clause by using both EXCEPT _________. a. subordinating conjunctions b. relative pronouns c. coordinating conjunctions.
349. TEACHING TECHNIQUES Select the option that contains the steps to be followed by the teacher when applying the manipulative strategy for teaching English. a. Identify concepts to be taught and ways to represent them; demonstrate the concept by using concrete objects; provide guided practice, give students time for additional practice; monitor and review. b. Identify key vocabulary; teach the activity; connect the vocabulary to past experiences; sort the words; plan activities or ways to use the words; perform the scenes by using the new vocabulary. c. Provide each student with a writing journal; demonstrate the process; have students draw and write; model writing based on a student’s picture and writing to encourage them; repeat the process daily.
350. TEACHING TECHNIQUES What strategy for the development of language is Miss Castillo using? In Miss. Castillo’s class, the students have some confusion understanding the word order of some grammatical structures. In order to help them, Miss Castillo has taken some sentences from collaborative retell and cut apart each word; then she has her students put sentence back together like a puzzle. After that, she asks to create new and more complex sentences as the students’ progress. a. Sorting activities b. Collecting and processing words c. Syntax surgery.
351. TEACHING TECHNIQUES Analyze the following case and identify the strategy that Mr. King is using: Mr. King is using story props to teach first graders the use of a Venn diagram. Two hula-hoops are laid on the floor and the students sit around them. In front of each student are three or four props from one of the two versions of Stone Soup that the class has read together in the past few days. Then he explains what a Venn diagram is through several activities where the students interact with the hula-hoops a. Total Physical Response b. Manipulative strategies c. Small groups and partners.
352. READING AND WRITING Read the article. Then use the information to choose the option that includes true statements. Becoming a Savant 1 Savants are people who have “superhuman” mental abilities. They might be able to remember every word of a book they just read, or quickly estimate the sum of a complicated math problem. This rare condition is often linked with autism. People with autism are socially disabled in terms of their interactions with others. Likewise, savants usually have difficulty assimilating with the rest of society. They have a hard time relating to everyone else. In a recent study, one savant said, “You’d think people would realize that we’re anxious and uncomfortable around them.” Another savant tried to explain his feelings by stating, “I’m not very flexible. I have my own schedule that I like to keep to, and I don’t care what people say.” 2 Emerging evidence in the past few decades has suggested that savant-like abilities can be related to injury to the left hemisphere of the brain. It can also be due to lack of development of this part of the brain. The right hemisphere of the brain compensates for this, and as a result, savants develop new mental abilities that are different from those of ordinary people. 3 Allan Snyder is a scientist in Sydney, Australia. He is known for his expertise in this area. Dr. Snyder decided to study the effect of electromagnetic pulses on different areas of the brain. He believed that he could bring out savant-like abilities in ordinary people by sending pulses to their brains. The study is an important one. If controlled correctly, people could certainly benefit from having these new abilities. 4 Dr. Snyder’s results were predictable, based on his theory. He found that the pulses transformed the way the brain handled information. People were able to recognize details that they never saw before in images and words. The longer they received the pulses, the more details they retained. For example, a New York Times reporter drew a cat several times while receiving pulses to his brain. He noted that each image was more and more detailed. “I could hardly recognize them as my own drawings, though I had watched myself render each one, in all its loving detail.” 5 Dr. Snyder’s persistence in this study has helped to give an idea of how a savant’s brain works. It has also shown that, with the right equipment, anyone can be a savant! a. Savants are different from everyone else because they can solve math problems. In Dr. Snyder’s study, the pulses made savants act like ordinary people Some people think that savants always want to be by themselves. b. Savants are different from everyone else because they can draw detailed pictures. In Dr. Snyder’s study, the pulses made people want to be savants. Some people think that savants have more abilities that are still hidden. c. Savants are different from everyone else because they have amazing mental abilities. In Dr. Snyder’s study, the pulses helped people notice more details. Some people think that savants are more like regular people than they are.
353. READING AND WRITING Read the introductory paragraph and choose the option that presents the correct information. (A) My brother is deaf, yet he is a very happy member of society. (B) While many hearing people at first feel sorry for him when they meet him, they only feel sorry for him because they don’t know him. (C) If they knew him like I do, they would know that he accepts his deafness, and he is proud to be a member of the deaf community. (D) Members of the hearing community would be wise to follow the lead of deaf people in defining their own groups, their own limitations, and their own acceptance of themselves. a. Sentence C states why the writer’s opinion matters. Sentence A hooks the reader with something surprising. Sentence D is the thesis statement. b. Sentence A states why the writer’s opinion matters. Sentence B hooks the reader with something surprising. Sentence D is the thesis statement. c. Sentence B states why the writer’s opinion matters. Sentence C hooks the reader with something surprising. Sentence A is the thesis statement.
354. READING AND WRITING Read the article. Then use the information to choose the option that includes true statements. Personalized Medicine 1 The human body consists of thousands of genes. When linked together, these genes make us the individuals we are. As medical technology advances, researchers have begun to switch from conventional studies to explore the idea of personalized medicine. This alternative medical approach involves aspects such as studying the genome, or complete gene make-up of an individual. The purpose is to find the likelihood of disease based on genetic risk factor and to interpret how cells react to disease. Then, doctors can prescribe the best drugs to treat individuals. 2 The idea of personalized medicine has its roots in the mid-1800s, when inherited traits were first discovered. As time went on, more discoveries were made. In the early 1900s, researchers began to connect genes with disease. People who had certain genes were recognized as more likely to get certain diseases. Early advocates may have begun to wonder, “Couldn’t we use this information to prescribe more effective drugs for individuals?” 3 And so the idea of personalized medicine was born, but it still had a long way to go. To understand the impact and importance of the human genome, the United States launched the Human Genome Project in 1990. Its goal was to identify all genes that exist in the human body. By the time the project completed in 2003, about 24,000 genes were discovered. Today, researchers continue to learn more about gene interaction and explore the idea of personalized medicine. 4 Patients are beginning to consult with their doctors to develop personalized gene mapping. Their aim is to discover likeliness for disease. Doctors then use this information to predict health issues and to prevent or treat them in ways that are most likely to work. 5 Personalized medicine is only just beginning to take its place in the medical environment, and it has elicited Mixed reactions. There are skeptical people with important questions, such as “How reliable are the results?” Still, the consensus is that personalized medicine has the potential to revolutionize healthcare. As research continues, the public will gain a better idea of whether this approach is truly beneficial. a. The main idea of the article is that personalized medicine is a medical approach based on genes. Personalized medicine works because it helps to find the best treatments for individuals. The Human Genome Project was a study that identified every human gene. b. The main idea of the article is that personalized medicine began in the 1800s and still exists today. Personalized medicine works because it shows doctors how to discuss medical advances. The Human Genome Project was a study that identified used human genes to create medicine. c. The main idea of the article is that personalized medicine is confusing for many doctors and patients. Personalized medicine works because it makes more medicines available for doctors. The Human Genome Project was a study that identified compared genes throughout history.
355. ACADEMIC WRITING Choose option that best explains the process to paraphrase correctly any information. a. Avoid using the original words, and sentence structure; make it a lot shorter than the original; and keep the meaning of the original. b. Use your own words, and own sentence structure; keep almost the same length as the original; and keep the meaning of the original. c. Use your own words, and own sentence structure; shorten the length from the original; and modify the meaning of the original.
356. ACADEMIC WRITING From the given example, identify the option that shows the correct form to cite a book according to APA style. a. Oshima, A. & Hogue, A. (2006). Writing Academic English . White Plains, NY: Pearson Education. b. Oshima, A. & Hogue, A. (2006). Writing Academic English. White Plains, NY: Pearson Education. c. Oshima and Hogue, A. (2006). Writing Academic English. White Plains, NY: Pearson Education.
357. INTRODUCTION TO APPLIED LINGUISTICS Examples of incorrect use of language such as “beautifuls flowers,” How many years old do you have?”, "I see many fishes” represent: a. Non -standard language b. Linguistic competence c. Language transfer.
358. Cuando un adulto hablante simplifica, modifica y adapta deliberadamente su discurso de acuerdo con la edad, el estado, la educación o los antecedentes culturales de un compañero de habla, o mescla dos o mas idiomas en el mismo discurso, este fenómeno se llama: a. non-standard language b. comportamiento del language c. code switching.
359. Most classic literature fits into the category of written words. Which of the following fits the same category? a. Songs and Oral Stories b. Pictures and Music c. Text and e-mails.
360. Literature is important to a cultural group because: It is a way to know exactly how everyone felt at that time. It always shows the hits and misses of the culture during that period It is a record of the ideas, beliefs and practices of the culture at that time.
361. Most classic literature fits into the category of written words. Which of the following also considered to be a classic literature? a. Songs and oral stories b. Pictures and Music c. Text and emails.
362. Drama and theatre are both closely related forms of storytelling that rely on language to tell a story, as well as: Technology, Paper and pencils. Costumes, setting and behaviour Music, dancing and lighting.
363. Storytelling has long been a form of literature and continuing cultural norms. Which of these is a modern form of storytelling, using today’s technology? a. Cellular phones. b. Pen and paper c. Facebook.
364. Literature is important to a culture group because___________. a. It is record of the culture at that time. b. It is a way to know exactly how everyone felt at that time. c. It always shows the best of the culture during that period.
364. Which of the following would be the best topic for an argument paragraph? Oranges are my favorite fruit. War can lead to many deaths. The Patriot act was a wise executive decision .
365. The Paragraph that tells a story is: a. Descriptive b. Narrative paragraph c. Expository paragraph .
366. The topic sentence of a paragraph should be__________. a. Be as broad and general as possible b. Be something the writer is interested in. c. It should be broad enough to explicate the topic/matter.
367. Which is an example of cause and effect sentence? a. Sam played video games and the internet slowed down. b. Sam used to play video games for hours, that is why his eye sight has gotten weaker. c. Sam bought new gaming console after Max did.
368. What are the two main reasons why paragraphs are used? a. To show a change of pace and to show when a new speaker begins talking b. To show a change of topic and to show when a new speaker begins talking. c. To show a change of pace and to show when a new person enters the room.
369. Read about the proposal to build a solar farm near a British village called Barnley. Then anwer the cuestios 6-9. The Environment. The Barnley Villaje Commite is opposed to plans to build a 6,890 panel solar farm on a 15 acre site adjacen to the village creacion ground, currently used for agriculture. Under the proposed scheme, area willbe surrounded by an 8ft-hight fence. The panels themselves willbe about 7 feet high. The commite has alrready lodged an appeal to the local authority against construction of the solar farm. the councillors are due to meet on 13ht to vote whether or not plans will go aheat. Local residents are invited to attend. Our objection will be presented before the board, and a representative from the solar firm SunGen will put forward the case for the development. Residents are encouraged to voice the objectipns to the development. These must address the aspects of the scheme that violate the current planning policy. However, you are welcome to make your objections personal, by stating how the plans will affect as a user of the recreation ground. Some of the most common objections are listed below: 1. The extensive views from the village and recreation ground across the open country will be bloked by the panels and high fencing. Furthermore, once the site has been build upon, it may be considered brownfield, thus an acceptable site for housing or industrial development. It does not, therefore comply with the local policy wich states that developments must not adversely effect on the appearance or character of the landscape. 2. The recreation ground has recently undergone major inprovements including a perimeter running trak, new playground equipment and seating. It is heavely used by families, sport teams, and dog walkers, and is regular used for village events. Cricket and football teams regularly use the recreation ground and it is not uncommon for balls to enter the field. Criketers are worried that they may become liable for damage to solar panels. If teams are forced to relocated, this would adversely affect the character of the village, and may jeopardise paeticipations of the children's teams. This goes against the National Planning Policy Framewok which requires developments to promote hight quality public space and encourage the active and continual use of public areas. 3. There has been no assessment of the extent to which noise from inverters and cooling fans will affected local residents. 4. As the ground beneath the solar panels will be surfaced , there will be more additional run-off of rainwater. The recreation ground already has problems with drainage, and these may be exaserbated by this development. A fromal flood risk assessment must be sumitted. 5. The lighting and security systems had been not outlined, it is not clear how the area will be made safe for children. Email your objections to planning@barnelycouncil.gov.uk, and quote the reference BLY7458/00578 in the subject line. What is the committee's opinion of the development? a. It is opposed to the development. b. It supports to the development. c. It is wainting for comments from residents before taking a viewpoint.
370. Regarding to the previous proposal reading. The solar farm would be built............... a. On the recreation ground b. In an agricultural field. c. On a brownfield site.
371. Regarding to the previous proposal reading. The meeting with councilors…. a. has already taken place. b. will take place shortly. c. has been proposed, but not planned.
373. Regarding to the previous proposal reading. Which of the following is NOT true of the proposed solar farm? a. I will be surrounded by a high fence. b. I will comprise of 6890 7-foot high panels. c. The solar panels will be placed directly on the grass.
374. Read the following book "Overview" In this series of lessons , student read newspapers articles obtained from newspapers websites. Students then identify journalism's "S Ws and 1H" (how, what when, where, why, and how) and complete a template with the corresponding information they have in the article. Finally, students use their notes to write a 20-wordssummary called a GIST. Once students have mastered writing a GIST using newspaper articles, the strategy is then applied to content areas text to support comprehension and summarizing skills". What does the writer intend to do? a. Presents a number of series b. Summarizes briefly what is contained in the book. c. Applies a new writing strategy.
375. Read and choose the best alternative. Different colors can affect us in many different ways; that's is according to Varity Allen. In her new TV series "Color me Healthy", Varity looks at the ways that colors can influence how hard we work and the choices we make. They can even change our emotions and even fluence how healthy we are. Have you ever noticed how peole always use the same colors for the same things?" says Verity. "Our toothpaste is always white or blue or maybe red. It's never green. Why not? For some reason we think that blue and white is clean, while we think of green products as being a bit disgusting.It's the same for businesses. We respect a company which writes its name in blue or black, but we don't respect one that uses pink or orange. People who design new products can use these ideas to influence what we buy." What is the writer doing in this reading selection? a. Reporting what happens in a new television series. b. Giving information about how colors influence us c. Giving information about a television presenter.
376. Read and choose the sentense in which the transition is correctly used. Different part of speaking are part of gender. As adults, men and women sometimes face difficulties in their communication with each other. Studies of communication show that if a woman tells her husband about a problem, she will spect him to listen and offer sympathy. She may be annoyed when he simply tells her how to solve the problem. Similarly, a husband may be annoyed when his wife wants to stop and ask a stranger for direction to a park or restaurant. ____________________________. a. His unlike wife he would rather use a map and find his way by himself. b. Unlike his wife, he would rather use a map and find his way by himself. c. His wife unlike, he would rather use a map and find his way by himself.
377. Read the following paragraph and choose the mest topic sentence. ________________________________. The lawyer promised to help her get U.S citizenship. She paid him more than a thousan dollars. At first, he said he was working hard to help her, but then suddenly, he disappeared. She never heard from him again, and she lost the mony. a. My sister was very angry and hurt b. My sister wanted to become a U.S. citizen c. My sister had a terrible experience with a fake lawyer.
378. Chomsky argued that children are________________ for language and that language develops in the child in just the same way that other biological functions develop. Mentally programmed Physically programmed Biologically programmed.
379. Innatist theory states that all human languages are fundamentally innate and they all share the same. a. Universal principles b. Universal grammar c. Universal Innatenes.
380. Sorry, but this chair is________. a. Me b. Mine c. My.
381. I____________________to the cinema. a. usually don’t go b. don’t usually go c. don’t go usually.
382. I eat pasta ________________week. a. twice in a week b. one time a week c. twice a week.
383. I don’t have _________free time. a. many b. any c. some.
384. A: __________to the cinema tomorrow. a. we will go b. do we go c. shall we go.
385. Choose the correct option to complete the sentence. “People ________ to watch movies at the mall” to be able is going to be able will be able.
386. We answer questions that the other private and government organizations________ able to answer. a. haven’t been always. b. haven’t always been c. have always not been.
387. Overly frugal people are addicted________. saving to save to saving.
388. vIf you think an answer is wrong. __________. a. Cross out it b. Cross it out c. Cross out.
389. Choose the correct option to complete the conversation. Scott: “It’s good to see you, too! Do you work here? Chris: “ yeah! I’ve worked at the post office_______________we graduated. What about you? a. ago b. for c. since.
390. Choose the correct option to complete the conversation. Tanisha: “We can start building the rocket as soon as we find some bottles . Would you mind looking for some?” Amber: “ I just looked. I _______ find any”. a. shouldn’t b, ‘d better not c. couldn’t.
391.Choose the correct option to complete the conversation. Leonard: “ Actually we’ve never been to this school before, so when we get there, we’ve got to look for the gym. I hope we get there soon. I have to do a lot of stuff this afternoon”. Mateo: “Maybe somebody already did everything for you and you_________ do it”. a. must not b. don’t have to c. can’t.
392. Choose the best answer for the following sentence. What should be ______ about the student lounge? a. do b. did c. done.
393. The topic sentence of a paragraph should be__________. a. Be as broad and general as possible b. Be something the writer is interested in. c. It should be broad enough to explicate the topic/matter.
394. The Paragraph that tells a story is________________. a. descriptive b. narrative paragraph c. expository paragraph.
395. Which of the following statements is not a characteristic of the inteactional perspective: a. Focuses on exploring the links between the stages of cognitive development and language skills. b. Linguistics structures will emerge only if theres is an already established cognitive foundation. c. Primary data is then used to make sentences or structures after a process of trial and error, correspond to those in adult speech.
396. Phonology__________. a. focuses on the sound that is produced when a person speaks. b. is concerned with the abstract, grammatical characterization of systems of sounds, it deals with the Systematic organization of sounds in languages. c. is the field of study devoted to the investigation of sound waves: it is the study of the physical properties of sounds?.
397. _____________________are vowels and consonants. a. Phonems b. Segmental phonems c. Suprasegmental phoneme.
398. In spoken English we have _____________________ 21 consonants and 5 vowels. 26 consonants and 5 vowels 24 consonants and 20 vowels.
399. When a person articulates a consonant sound__________. a. there is friction in the flow of air expelled through the oral cavity. b. there is no friction in the flow of air expelled through the oral cavity. c. there is friction or no depending of the consonant.
400. English is a __________ in a community where another language is spoken. a. Foreign language b. Native language c. Second language.
401. ________ deals with how sounds are put together to form words, and everything related to word formation. a. Semantics meaning of words sentences and texts b. Pragmatics how people comprenhen and produce a communicative act or speech act in a concrete speech situation c. Morphology.
402. Accent is___________ a. a kind of language that does not follow the grammar of pronunciation rules of standard language. b.the form or variety of a spoken language peculiar to a region, community or social group. c. a regional variation of language which does not differ in grammar and vocabulary used by a group of speakers.
403. The “universal grammar” theory implies that: ________________. a. the grammar of any language is commonly approached in a descriptive and prescriptive way. b. all humans create their own set of grammar rules to be able to communicate properly. c. all human beings have an inner ability to use grammar rules in their native language.
404. It constitutes a slightly different variation of a language, similar to dialect but determined by the subject matter rather than geographical and social issues. For example: “Baby-talk” a. Dialect b. Accent c. Register.
405. The deliberate change from one manner or style of speaking to another is normally called> a. Dialecto b. Code switching c. Motherese.
406. Applied linguistics uses information from Sociology, Psychology, Anthropology, and information theory as well as from linguistics in order to develop: a. A communicative act of speech b. Theoretical models of language and language use. c. Social class difference.
407. Please choose the correct answer from the choices presented. Regarding English formation and development, this language has been influenced by………. a. Germanic and Latin languages b. Asian Languages c. Native American Languages.
408. When English language is used in regional and national administration, in mass media, in educational system as a means of instruction, and is taught to immigrants, it is known as a: a. Second language (ESL). b. Foreign language (EFL). c. Native language.
409. The presence of Universal Grammar in the brains of children allows them to deduce the structure of their native languages from “more exposure”, is a theory of language acquisition attributed to ________________. a. Bloomfield. b. Krashen. c. Chomsky.
410. Anthropology studies________________________. a. Human social behavior and the study of society b. The origin and behavior of man, including the physical, asocial and cultural development. c. The study of the nature, structure and variations of language.
Denunciar test Consentimiento Condiciones de uso